Docsity
Docsity

Prepare for your exams
Prepare for your exams

Study with the several resources on Docsity


Earn points to download
Earn points to download

Earn points by helping other students or get them with a premium plan


Guidelines and tips
Guidelines and tips

NSG 6440 Pre Predictor Test Questions And Answers With Comprehensive Explanation, Best Exa, Exams of Nursing

NSG 6440 Pre Predictor Test Questions And Answers With Comprehensive Explanation, Best Exam Solution 2022/2023 Update RATED A+ South University, Savannah

Typology: Exams

2022/2023

Available from 10/07/2022

hesigrader002
hesigrader002 🇺🇸

4.1

(38)

1.7K documents

1 / 91

Toggle sidebar

Partial preview of the text

Download NSG 6440 Pre Predictor Test Questions And Answers With Comprehensive Explanation, Best Exa and more Exams Nursing in PDF only on Docsity! 1 NSG 6440 Pre Predictor Test Questions And Answers With Comprehensive Explanation, Best Exam Solution 2022/2023 Update RATED A+ South University, Savannah Typed Pre-Predictor 1) A 15 years old high school student with a mild sore throat and low-grade fever that has persisted for about 3 weeks. She reports general malaise, fatigue, and loss of appetite. The NP suspects mononucleosis. Which of the following is the LEAST appropriate intervention? a. Palpate the lymph nodes and spleen b. Examine the posterior oropharynx for petechiae c. Obtain a CBC, throat culture, and heterophil antibody test. d. Obtain an urinalyses and serum for LFTs and amylase Explanation: mononucleosis is a symptomatic infection caused by the Epstein-Bar virus. Common is people 15-24 years of age. Common signs and symptoms following incubation period (1-2 months) include fatigue, chills, malaise, anorexia, white tonsillar exudates and lymphadenopathy or posterior cervical region. Splenomegaly can be present. A maculopapular or occasionally a petechial rash occurs in less than 15% of patients. A diagnosis is usually made using the Monospot. In addition, neutropenia and lymphocytosis are usually detected in the CBC. 2) A 32 years old male patient complaint of urinary frequency and burning on urination for 3 days. Urinalyses reveals bacteriuria and positive nitrites. He denies any past hx. Of urinary tract infections. The initial treatment should be: a. trimethoprim-sulfamethoxazole (Bactrim, Sulfatrim) for 7-10 day b. ciprofloxacin (Cipro) for 3-5 days c. Trimethoprim-Sulfamethoxazole for 3 days d. 750 mg ciprofloxacin as a one-time dose Explanation: trimethoprim-sulfamethoxazole (TMPS) is usually n appropriate medication to treat urinary tract infections in most patients. In the case of community resistance to TMPS >20%^, another medication should be substituted. In men, the appropriate length of time is 7-10 days. Women may be treated for 3 days for uncomplicated UTI 3) Which agent is most effective for the treatment of nodulocystic acne? a. Benzoyl peroxide (Benzac) b. Retinoic acid (Retin A) c. Topical tetracycline d. Isotretinoin) 2 Explanation: Isotretinoin (Accutane) is a systemic agent indicated for treatment with severe inflammatory acne. Guidelines for its use must be clearly understood by the patient. A woman of childbearing age must use an effective method of contraception because isotretinoin is teratogenic. There are many restrictions in prescribing this medication because of the teratogenic effects is given during pregnancy. Therefore, it is a pregnancy category X. 4) An 18 y/o woman is taking a combined hormonal oral contraceptive. She should be instructed to use a backup method for the prevention of pregnancy a. Throughout the week of placebo pills b. If prescribed topiramate (Topamax) for the treatment of migraines. c. If prescribed amoxicillin/clavulanate (Augmentin) for a sinus infection d. if she forgets to take a single dose of the contraceptive Explanation: Anticonvulsant including phenytoin (Dilantin), carbamazepine (Tegretol), primidone (Mysoline), topiramate (Topamax) and oxcarbazepine (Trileptal) reduce the effectiveness of contraceptives. Depo-medroxyprogesterone acetate injections or levonorgestrel- releasing intrauterine devices would be a better method of contraceptive for patients taking anticonvulsants. Most commonly used antibiotics have not been proven to reduce the effectiveness of contraceptives. Rifampin is an exception, and additional …. Be used by women taking this drug and using oral contraceptives, transdermal, or vaginal ring preparations. Additional backup contraception should be used if taking antifungal agents. No additional protection is needed thought the week of placebo pills. Missing one single dose of contraceptive does not require additional protection, missing more than one doses does. 5) A 44 years old female patient has diabetes. Her total cholesterol (TC) is 250 mg/dl (6.5 mmol/L), LDL= 190 mg/dL (4.94 mmol/L), HDL= 25 mg/dL (65 mmol/L), and triglycerides= 344 mg/dL (8.94 mmol/L). What agent have the greatest effect on improving her lipid profile and reducing morbidity and mortality associates with dyslipidemia? a. Niacin (Niaspan) b. Atorvastatin c. Omega 3 fatty acids d. Fenofibrates Explanation: First and foremost, it is essential to educate individuals on a heart-healthy lifestyle. LDL-C is one of the major culprits in the development of atherosclerotic heart disease. The target level of LDL-C is between 50 to 70mg/dl to prevent plaque formation in the blood vessels. Guidelines strongly recommend statin therapy because they primarily lower LDL-C levels, but they also have the secondary effects of lowering triglyceride and increasing HDL-C levels. 6) A 30 years old female comes into a clinic with classic signs and symptoms of appendicitis. The NP fails to refer the patient to a surgeon. The appendix ruptures and the woman die. This is an example of a. Failure of diligence b. Professional liability c. Negligence 5 Macrovascular complications: Coronary artery disease, Cerebrovascular disease, Peripheral vascular disease 14) A 6 y/o presents w/ complaints of sore throat and fever for 2 days. He has multiple vesiculated ulcerations on his tonsils and uvula. There are no other remarkable findings. What is the most likely diagnosis? a. Viral pharyngitis b. Herpangina c. Epiglottitis d. Tonsillitis Explanation: Herpangina is a viral infection common in toddlers and young children caused by Coxsackie virus. The clinical findings of numerous, small (1-2 mm) ulcerations on the tonsils and uvula are typical of herpangina. The ulcerations can be very painful but usually resolve in 7 to 10 days. Treatment is symptomatic. 15) A patient has Kawasaki syndrome. Which characteristics would be UNUSUAL? a. Age > 15 years b. Fever > 101 F (38.3 C) c. Exudative pharyngitis d. Painful rash explanation: Kawasaki disease is an acute febrile vasculitis syndrome that evolves inflammation of the blood vessels. This condition often causes cardiac complication in children by damaging the coronary arteries is most prevalent in children of Asian ethnicity. Diagnosis of Kawasaki disease requires presentation of fever and 4 of the following criteria: bilateral bulbar conjunctival injection, oral mucous membrane changes, peripheral extremity changes, polymorphous rash, and cervical lymphadenopathy. 16) According to the JNC 8 guideline hypertension in a 40 y/o can be diagnosed when blood pressure exceeds a. 140/90 b. 130/90 c. 125/85 d. 150/100 Explanation: According to JNC 8 guidelines, hypertension is a sustained elevation of systolic BP greater than or equal to 140 mmHg or diastolic BP greater or equal than 90mmHg, taken from 2 or more readings on 2 different occasions after an initial screening. 17) A 48 y/o female complains of pain and stiffness in her right hip and knee that is mild on awakening in the morning, get worse as the day progresses and is relived with hot baths and ibuprofen. Crepitus is palpated on range of motion of the knee. Signs of inflammation are notably absent. What is the most likely diagnosis? a. Rheumatoid arthritis (RA) b. Gout c. Osteoarthritis (OA) d. Osteoporosis Explanation: Rheumatoid arthritis is characterized by several joint deformities, usually 6 bilaterally symmetrical. RA is characterized by inflammatory processes, while OA is not. RA and OA are chronic conditions. Gout is characterized by acute exacerbations related to a defect in purine metabolism, increased uric acid production, or decreased uric acid excretion. 18) A 7-year old female patient presents with severe injuries that are inconsistent with the explanation given for them. The nurse practitioner questions the mother about abuse. She admits that her husband, the child’s father, beat the child. How should the nurse practitioner proceed? a. Inform the mother that the abuse must be reported to child protection authorities. b. Counsel the mother that if it happens again it will be reported to child protection service. c. Ask the child what she did to cause the punishment d. Refer the family to the National Domestic Violence hotline. Explanation: An awareness of all the unique presentations of child abuse, subjective or objective, physical, sexual, or neglect, is essential for NPs. Objective findings, such as broken bones, burns, and bruises, may be as a result of an intentional injury from physical abuse or from an unintentional injury as a result of neglect. Both must be considered. Repeated visits to the emergency department, frequent or suspicious injuries, or bilateral or multiple healing fractures are often indicators of physical abuse. Once the diagnosis of child abuse has been made, the primary role of the NP is to ensure the child is safe. If the NP suspect that a child is undergoing abuse, it’s critical to report it—and to continue reporting each separate incidence if it continues to recur. 19) A 1-month-old presents with reported recurrent diarrhea, screaming, and drawing up of the legs followed by periods of lethargy. On physical examination, a “sausage-like” mass in the upper right quadrant of the distended abdomen. Which of the following is the most likely diagnosis? a. Intussusception b. Volvulus c. Crohn’s disease d. Foreign body in the GI tract explanation: Intussusception is one of the most common causes of abdominal obstruction in children prior 2 years of age; is best described as a portion of the intestine which telescopes into a more distal intestinal segment. The classic triad of intussusception include crampy (intermittent, also known as colicky) abdominal pain, vomiting, and bloody stools. The patient may pull up his knees with crying. The patient may develop vomiting with bilious emesis. Progressive lethargy/altered level of consciousness and pallor is common. The etiology of this lethargic presentation is not known, but it tends to occur in younger infants. As intussusception progresses, a palpable, sausage-shape mass may develop. Some hypothesize that this is due to release of endogenous opioids or endotoxins released from ischemic bowel. Intussusception in a child presenting with lethargy is often difficult to diagnose since other causes of lethargy such as dehydration, hypoglycemia, sepsis, toxic ingestion, post-ictal state, etc., must also be considered. Ultrasound is the preferred diagnostic test. Enemas is considered the first line of treatment prior surgery. Volvulus occurs more frequently in middle-aged and elderly men. Cronos’s is most often diagnosed between 13 – 30 years of age. 20) A middle-aged female complains of insomnia, night sweats, feeling intensely hot, emotional 7 lability, extreme nervousness and impatience. The LEAST likely cause of her symptoms is a. Thyrotoxicosis b. Menopausal vasomotor instability c. Alcohol or another drug withdrawal d. New onset type 2 diabetes mellitus Explanation: New onset diabetes produces elevated serum glucose levels less than 200 mg/dL (7.0 mmol/L) and usually no clinical signs or symptoms. At higher levels, the patient may report lethargy, fatigue, weakness, weight loss, and polydipsia, polyuria, and/or polyphagia. Complaints of insomnia, night sweats, feeling intensely hot, emotional lability, and extreme nervousness may be caused by thyroxine excess, menopausal instability, or withdrawal from alcohol or other drugs of addiction. 21) Which of the following indicated need for further evaluation? a. A 7 y/o girl with vaginal bleeding b. A 7 years old girl with no true pubic hair c. A 12 years old boy with sparse, slightly pigmented pubic hair d. A 12-year-old girl with breast buds explanation: In newborn girls, withdrawal bleeding may occur as a normal response to maternal estrogen leaving the infant’s uterine lining. Vaginal bleeding after the first few weeks of life and before puberty is considered abnormal. Early vaginal bleeding can indicate trauma, foreign bodies (toilet paper is the most common), vulvovaginitis, hemangiomas, benign polyps, precocious puberty, or sexual abuse. 22) A young couple is being seen by the NP for preconception counseling. They express a wish for pregnancy within the next 3 month and are very eager to know what they can do now to “make the baby as healthy as possible”. Which of the following should the NP encourage to decrease the chance of neural tube defect in the fetus? a. Maternal alpha-fetoprotein level b. Folic acid 0.4 mg daily c. Rubella vaccine today d. Vitamin E 400 IU daily Explanation: Supplementation with folic acid decreases the development of neural tube defects, such as spina bifida and anencephaly. Folic acid plays an essential role in neural tube closure. Neural tube development/closure takes place in the fist 4 weeks of embryonic life (6 weeks’ gestation). U.S. Public Health Service and the CDC recommend that all women of childbearing age consume 0.4 mg of folic acid daily to prevent tube defects. 23) A patient with a past history of documented coronary arterial blockage less than 70% complains of chest pain several time p…. which is relived with nitroglycerin. Which is the most appropriate initial action for the NP? a. Refer to a cardiologist as soon as possible b. Prescribe long-acting nitroglycerin c. Order treadmill stress test d. Prescribe an ACE inhibitor and re-evaluate in 24 to 48 hours Explanation: The patient has at least a 70% occlusion of a major coronary artery. For a patient to be considered for CABG, the coronary arteries to be bypassed must have 10 for further diagnostic testing. Diagnostic testing procedures should proceed any surgical intervention like fundoplication. 32) A 51 –year- old post-menopausal female, request guidance regarding osteoporosis risk. The NP would be correct to recommend all of the following EXCEPT a. Moderate weight bearing exercise 3 times per week b. 1200 – 1500 mg calcium daily c. Avoidance of alcoholic beverages d. Weight loss Explanation: the recommended calcium intake is 1,200 mg – 1500 mg a day. Regular weight- bearing and muscle-strengthening exercise reduce the risk of falls and fractures. Moderate alcohol intake has no known negative effect on bone and may even be associated with slightly higher bone density and lower risk of fracture in postmenopausal women. However, alcohol intake of more than two drinks per day for women or three drinks a day for men may be detrimental to bone health, increases the risk of falling and requires further evaluation for possible alcoholism. In addition, Risk factors for osteoporosis include age greater than 65, white or Asian ethnicity, cigarette smoking, inactive lifestyle, low weight and postmenopausal estrogen deficiency including premature menopause. 33) According to Erickson, the developmental task of the elderly adult is: a. Intimacy VS isolation b. Ego integrity VS despair c. Industry VS self-doubt d. Trust Vs mistrust explanation: 11 34) Moderate weight loss, particularly of visceral adipose tissue in patients with type 2 diabetes mellitus may have all of the following beneficial effects EXCEPT: a. Improved insulin sensitivity b. Increased glucose uptake and utilization by the cells c. Increase lean muscle mass’ d. Improved lipid profile Explanation: Regular, consistent exercise is an essential part of diabetes and prediabetes management. The ADA recommends that people with diabetes perform at least 30 minutes, 5 days/wk of a moderate-intensity aerobic physical activity. The ADA also encourages people with type 2 diabetes to perform resistance training 3Xwk in the absence of contraindications. Exercise contributes to weight loss, which further decreases insulin resistance. The therapeutic benefits of regular physical activity may result in a decreased need for diabetes medications to reach target blood glucose goals. Regular exercise may also help reduce triglyceride and low-density lipoprotein (LDL) cholesterol levels, increase high-density lipoprotein (HDL), reduce BP, and improve circulation. Patients who use insulin, sulfonylureas, or meglitinides are at increased risk for hypoglycemia when they increase physical activity. 35) A 12 y/o presents with eat pain or 36 hours duration. The NP diagnoses acute otitis media because the a. Tympanic membrane is bulging and glossy with tiny bubbles visible posteriorly b. Tympanic membrane is retracted against boy landmarks c. Bony landmarks are obscured, and the tympanic membrane is mildly erythematous, dulls and immobile. d. Canal is narrowed, erythematous, and exquisitely tender with speculum contact Explanation: Serous otitis media typically presents with a flat or bulging and tympanic membrane with a fluid line and/or tiny bubbles visible posteriorly. The tympanic membrane may be immobile and retracted against the bony landmarks when the eustachian tube is swollen or congested as with the common cold or allergies. Narrowing of the external canal with erythema and extreme tenderness of the canal wall is indicative of otitis externa. 36) The NP should instruct the mother of an infant with thrush to: a. Take oral nystatin since she is breastfeeding b. Stop breast feeding until the thrush has resolved c. administer antifungal medication to the infant prior to feeding d. sterilized pacifiers and bottle nipples explanation: Most bouts of oral thrush occur for no apparent reason. However, regularly sterilization of pacifiers, bottles-feed and other mouth toys used by the baby may help to prevent some bouts. 37) A 26 y/o patient, 18 weeks pregnant with twins, has been healthy and has followed recommendations of her nurse midwife. She is in the office to discuss results of her maternal serum alpha-fetoprotein (MSAFP) test. which show elevation in this particular pregnancy? a. Elevated MSAFPO is an indicator of Down Syndrome 12 b. Low MSAFP is expected at 18 weeks’ gestation c. Neural tube defects are highly probable d. Elevated MSAFP is an expected finding Explanation: elevated maternal serum alfa Protein is expected when a woman is pregnant with twins. An elevation can also be an indication of a miscalculated due date. Low levels can indicate Down Syndrome 15 b. Chronic cholecystitis c. Acute pancreatitis d. Myocarditis Explanation: The “typical” patient with chronic cholecystitis is “female, fat, fertile, and (over) 40” years of age. Myocarditis produces retrosternal pain that is not related to diet. The pain of pancreatitis radiates to the back. The pain of hepatitis is non-specific upper quadrant pain. 42) A 16 y/o male presents w/ mild sore throat, fever, fatigue, posterior cervical adenopathy, and palatine petechia. With for this patient, what drug would be the LEAST appropriate to prescribe? a. Ibuprofen b. Erythromycin c. Ampicillin d. Doxycycline Explanation: Mono can be managed with supportive care, including NSAIDs, and warm sailing gargles. patients with strep throat should be prescribed penny ceiling or every throw myosin instead of amoxicillin or ampicillin because amoxicillin and ampicillin often cause a rash in mono patients. 43) A characteristic of delirium that is typically absent in dementia is a. Acute onset of confusion in a previously alert and oriented patient b. Gradual loss of short-term memory c. Loss of language skills d. Long term memory gaps filled in with confabulation Explanation: Delirium is acute confusional state, is potentially a reversible cognitive impairment that often has a physiological cause; physiological causes include electrolyte imbalances, cerebral anoxia (an absence of oxygen); hypoglycemia; medication effects; tumors; subdural hematoma; and cerebral infection, infarction, or hemorrhage. Dementia is a generalized impairment of intellectual functioning that interferes with social and occupational functioning. It is an umbrella term that includes Alzheimer's disease, Lewy body disease, frontal-temporal dementia, and vascular dementia. 44) It is imperative that the NP teach patients taking oral contraceptives to report any of the danger signs of complications. Which of the following would be the LEAST concern to the NP? a. Lower leg pain b. Upper abdominal pain c. Chest pain d. Weight gain Explanation: Common side effects of OCP includes: bleeding between periods, nausea, breast tenderness, headaches and weight gain. The symptoms often subside after a few months of use. Lower Leg pain can be indicative of- DVT. Other major complication can include the liver, gallbladder or the cardiac organ. 45) Which of the following oral medications should be avoided in a child under 8 years of age? 16 a. Cephalexin (Keflex) b. Tetracycline c. Rifampin d. Metronidazole (flaggyl) Explanation: Children younger than 8 years old should not take tetracycline. Deposition in the bone and primary dentition occurs during calcification in growing children. This causes discoloration and hypoplasia of the teeth and a temporary stunting of growth. 46) Which of the following descriptions of Denver II Developmental Screening Test is most accurate? a. Applicable to children from birth to 2 years; evaluates four major categories of development: motor, intellectual, emotional, and language to determine whether a child is within normal range for various behaviors or is developmentally delayed b. Applicable to children from birth to 5 years; evaluates four major categories of development: motor, vision, hearing and psychosocial to determine whether a child is normal or developmentally compromised c. Applicable to children from age 6 months to 6 years; evaluates 4 major categories of development: intellectual, verbal, social and memory to determine IQ and aptitude d. Applicable to children from birth to 6 years; evaluates four major categories of development: gross motor, fine motor-adaptive, language, and personal social to determine whether a child is within normal range for behaviors or is developmentally delayed. Explanation: The Denver Developmental Screening Test (DDST) is a screening test for cognitive and behavioral problems in children from birth until the age of 6 The purposes of the DDST and Denver II are to screen children or possible developmental problems and to verify suspected problems with an objective measure. The Denver II screens general development in four areas: Social – Personal: Aspects of socialization inside and outside the home – eg, smiling. Fine motor function – Adaptive: Eye/hand co-ordination, problem solving and manipulation of small objects – eg, grasping and drawing. Language: Production of sounds, and ability to recognize, understand and use language – eg, ability to combine words Gross motor functions: Motor control, sitting, walking, jumping, and overall large muscle movements. 47) A 12-month-old has conjunctivitis in his right eye with a mucopurulent discharge. The mother asks if the child can forego for antibiotic eye drops because he does not’s like drops in his eyes. The NP replies that: a. If untreated, conjunctivitis may permanently damage the cornea b. Conjunctivitis is usually caused by a virus. Let’s wait a few days c. If no one else at the daycare has it, we can wait a few days. d. An oral antihistamine may be prescribed instead of eye drops. Explanation: Pink eye does not normally cause loss of vision, especially when treatment is administered in a timely manner. However, if bacterial conjunctivitis is serious and left untreated, it may damage the eye permanently, leading to loss of vision. 17 48) which oral hypoglycemic agent would be safest for an elderly patient if hypoglycemia is a major concern? a. Tolbutamide (orinase) b. Glipizide (Glucotrol) c. Metformin (Glucophage) d. Chlorpropamide (diabinese) Explanation: When used alone, metformin typically does not cause hypoglycemia and is usually weight neutral or causes modest weight loss. Given the low risk of hypoglycemia, metformin has an important role in elderly patients with diabetes, as long as patients are selected appropriately to avoid the risk of lactic acidosis. 49) the components of the Denver II Developmental screening test are: a. Personal/social, fine motor, gross motor, language b. Intelligence, motor performance, language development c. Vocabulary, clarity of speech, abstract thinking d. Problem-solving, speech, gross motor, fine motor Explanation: The Denver II Developmental Screening Test is the most widely used tool to assess early childhood development and rates the categories of personal–social, fine motor– adaptive, language, and gross motor skills. The nurse should explain to the parent before administering the test that this test does not measure intelligence but of the child's level of development or ability to perform age-appropriate tasks. DDST II Assesses: Gross motor function Language development Fine motor-adaptive skills Personal-social skills 50) A patient has been taking fluoxetine (prozar) since being diagnosed with major depression 7 month ago. She reports considerable empowerment in her symptoms and her intention to discontinue the medication what should be the NP’s recommendation? a. Advice the patient to stop the antidepressant medication b. Question the patient to determine if the self-assessment is correct before advising her to discontinue the medication c. Recommend that the patient continue the antidepressant medication for at least 4 more month d. Discuss with the patient that need to take the antidepressant medication indefinitely Explanation: Anxiety Treatment Clinical guidelines for managing MDD also recommend that patients should maintain antidepressant use for at least 6–9 months after full symptom remission and that patients with some risk factors. 51) for the general adult population, total dietary fat intake should be no more than what percent of total calories? 20 56) what is a Tzanck test used to diagnosed? a. Mastoiditis b. Trichomoniasis c. Hyphema d. Herpes virus Explanation: Tzanck prep is the only test in this list which is diagnostic for herpes simplex. KOH prep is used to diagnose candidal and bacterial vaginosis. Gram stain is used to help distinguish Gram positive and Gram negative organisms. The Papanicolaou (Pap) smear is used to screen for cervical dysplasia and cancer. 57) a 40 y/o female with hx. Of frequent sun exposure present with multicolored lesion on her back it has irregular borders and is about 11 mm in diameter. What should the NP suspect? a. Squamous cell carcinoma b. Malignant melanoma c. Common Nevis d. Basal cell carcinoma Explanation: ABCDE RULE Another commonly used tool for early detection of melanoma is the ABCDE acronym (Asymmetry, Border irregularity, Color variation, Diameter >6mm and Evolution or history of change) introduced to alert patients and health professionals to the diagnosis of melanoma. 58) a 16 y/o sexually active female presents to the clinic. She has never had vaccinations for hepatitis A or B. She has had one MMR immunization, and her last tetanus vaccination was 4 years ago. which vaccination would be contraindicated without further testing? a. Hepatitis B b. Hepatitis A c. MMR d. Tetanus Explanation: Patient should have a pregnancy test before MMR vaccine. The MMR vaccine is contraindicated during pregnancy because it contains live, attenuated viruses that pose a theoretical risk to the fetus 59) following the finding of prostate gland abnormalities on DRE, The NP orders appropriate labs. When preparing to review with the patient, the NP knows all of the following are true EXCEPT a. Normal PSA is 10 mg/ml or less b. PSA is elevated in the presence of malignant prostate epithelium c. PSA is elevated in the presence of BPH d. Positive serum acid phosphatase reflects malignancy of the prostate gland with bone metastasis. Explanation: The effect of digital rectal examination (DRE) on serum PSA levels is still debatable.....Although, there was a statistically significant increase in serum PSA levels 30 min after DRE, the clinical significance of this increase in PSA values with a mean difference of 0.4 ng/ml remains to be clarified. 21 60) in which of the following presentations is further diagnostic testing NOT warranted? a. Bilateral gynecomastia in a prepubertal male of average weight; Tanner stage 1 b. Bilateral gynecomastia in a 13 y/o male with normal testicular size and volume c. Recent onset gynecomastia in a 20-year-old male with breast tenderness d. Unilateral breast mass which is 5 centimeters in diameter Explanation: Gynecomastia is the benign enlargement of male breast glandular tissue and is the most common breast condition in males. Pubertal gynecomastia usually begins at age 10-12- years-old and peaks at ages 13-14.https://www.ncbi.nlm.nih.gov/pmc/articles/PMC3987263/ 61) a 59 years old post-menopausal woman has atrophic vaginitis. She has a hx. Of breast cancer at age 40 years. What is the appropriate initial treatment for this patient? a. Oran conjugated estrogens b. Oral medroxyprogesterone aetate c. Topical medroxyprogesterone aetate d. Topical conjugate estrogen cream Explanation: PREMARIN (conjugated estrogens) Vaginal Cream is a mixture of estrogens indicated for: Treatment of Atrophic Vaginitis and Kraurosis Vulvae Treatment of Moderate to Severe Dyspareunia, a Symptom of Vulvar and Vaginal Atrophy, due to Menopause. https://www.pfizermedicalinformation.com/en-us/premarin-vaginal-cream 62) Advances in obstetric and neonatal care have a. Helped to identify the cause of cerebral palsy b. Demonstrated the cerebral palsy is a direct result of birth asphyxia c. Has no effect on the incident of cerebral palsy d. Resulted in a dramatic decrease in the incidence of cerebral palsy Explanation: Cerebral palsy is a common, non-progressive, encephalopathy that is believed to be from a defect in the developing brain. Improvements in perinatal care have not affected the incidence. 63) which of the following physical modalities recommended for treatment of rheumatic arthritis provides the most effective long-term pain relief? a. Superficial and deep heat b. Application of cold c. Transcutaneous electrical nerve stimulation (TENS) d. Exercise Explanation: exercise is most consistently effective in reducing the pain associated with rheumatoid arthritis. Exercise improve blood flow, cartilage health, range of motion. Exercise can also improve self-efficacy. Patients with RA should be cautioned to limit joint range of motion, and/or splint affected joints during acute…..Heat, cold, and TENS application can also have a role in pain relief. 64) The NP is counseling a 25 y/o sexually active male patient about condom use, which of the following statement is INCORRECT a. Adequate lubrication is needed to prevent damage to the condom b. Roll the condom over an erect penis before any sexual contact 22 c. Make sure the condom is tight against the head of the penis d. Withdraw while the penis is erect, so that the condom stays in place Explanation: Condoms come in all different types and sizes. Condoms that are too tight can choke off blood supply, which contributes to erectile dysfunction. And if you choose a condom that’s too big, it can slip off during intercourse, which not only defeats the purpose of wearing the condom but can also undo an erection. 65) a patient with moderate persistent asthma will probably be most effectively managed with daily a. Oral leukotriene blockers b. Short and long acting bronchodilators c. Inhaled steroids and long acting bronchodilators d. Oral steroids and short acting bronchodilators Explanation: People with moderate persistent asthma are not well controlled on low doses of inhaled corticosteroids. A combination of this drug and long-acting inhaled beta2 agonists provides improved control compared with doubling of the maintenance dose of inhaled corticosteroids. https://www.ncbi.nlm.nih.gov/pubmed/16935690 66) diagnosis of systemic lupus erythematosus (SLE) is made a. On the basis of demonstrable anti-nuclear antibodies (ANA) b. Considering symptom complex with confirmation by laboratory test c. Using renal function studies and rheumatoid factor for confirmation d. On the basis of renal or cutaneous biopsy Explanation: The American College of Rheumatology has proposed the revised criteria for diagnosis of systemic lupus erythematosus (SLE) abnormalities associated with disease. At least four of these criteria are required to make the diagnosis. Laboratory test…. 67) which of the following would not be considered a developmental red flag to a NP assessing a 4 y/o? a. Persistent fear of going to sleep b. Missing speech sounds c. Fire-setting d. An imaginary friend Explanation: The parents need to understand that the child's behavior is not uncommon. Imaginary friends are common in the preschool-age child. The child's behavior is not abnormal. The child does not have a deep-set psychological need. The child is not at risk for social isolation. 68) A 44 y/o patient complains of stuffiness and soreness in his hands, hips and knees. There is noticeable PIP and DIP joint enlargement in his hands.. The NP suspects arthritis. All of the following questions are helpful in differentiation between rheumatoid arthritis (RA) and osteoarthritis (OA) EXCEPT a. on the joint of your fingers never feel particularly warm or hot? b. tell me about your usual energy level 25 b. legalized abortion, frequent viewing of adult television and movies, and lack of church affiliation c. lower socioeconomic status, use of tobacco, alcohol or other drugs, and single parent household d. exposure to sex education information at school Explanation: Factors found to be associated with early sexual activity are lower socioeconomic status, use of tobacco, alcohol, or other drugs, and belonging to a single parent household. 75) the finding which is most consistent with a diagnosis of benign prostatic hyperplasia is digital palpation of prostatic gland that is a. enlarged, symmetrical, semi firm and non-tender b. enlarged, symmetrical, boggy and exquisitely tender c. asymmetrical and nodular d. exquisitely tender with absence of median sulcus Explanation: During the digital rectal exam, the practitioner palpates the prostate, noting its size, shape, and consistency and any nodules or tenderness. The normal prostate is rubbery and nontender. With BPH, the prostate feels symmetrically enlarged, firm, slightly elastic, and smooth. A distinct, hard nodule may signal a malignancy. 76) The NP correctly diagnoses iron deficiency anemia in a female patient whose lab report reveals a. an increase reticulocyte count b. a mean corpuscular volume (MCV) > 100 c. hemoglobin (hgb) 140 mg/dL (140 g/L) d. an increased total iron binding capacity (TIBC) Explanation: A total iron binding capacity value above 450 mcg/dL usually means that there's a low level of iron in your blood. This may be caused by a lack of iron in the diet, increased blood loss during menstruation, pregnancy, or a chronic infection. 77) Strabismus is observed in a 13-month-old child. The most appropriate action for the NP to take a. refer the patient to an ophthalmologist b. patch the child affected eye c. follow the child closely for 2 more month d. teach the patient and parent, eye muscle exercises Explanation: Strabismus can also be an indicator of severe eye and health problems. As it can indicate poor vision, it may in rare cases be the first sign of childhood cataract, glaucoma, or tumors of the eye, optic nerve, orbit or brain, such as retinoblastoma, glioma, or rhabdomyosarcoma. 78) giving appropriate patient education by the primary care provider, poor compliance with medical recommendations is most often due to: a. willful disobedience b. vision and/or hearing deficits c. anxiety d. limited cognitive ability 26 Explanation: In the National adult Literacy Survey, 75% of Americans with chronic or mental health problems scored in the lowest 2 out of 5 …. Literacy. Among the elderly, minorities, and lo income groups, 2/5 cannot read the simple brochure. Most patients are not likely to volume….. 79) the physical exam findings in an infant are most effected by which item listed below? a. what was the Apgar score at one minute? b. is the child breast of bottle fed? c. are the immunizations current? d. what was the gestational age at birth? Explanation: Gestational age is the most important predictor of growth impairment, followed by birth weight and length standard deviation scores. https://www.sciencedirect.com/topics/nursing- and-health-professions/gestational-age 80) Babies should begin oral iron supplementation: a. when they are able to eat iron enriched cereal b. 2 months of age c. 4-6 months d. any time after their teeth have erupted Explanation: Recommends giving breastfed infants 1 mg/kg/day of a liquid iron supplement until iron-containing solid foods are introduced at about 6 months of age. 81) the most common cause of cerebral palsy is a. unknown b. intra-partial hypoxia c. in utero infection d. intraventricular hemorrhage Explanation: in approx. 70% of cases, neither cause (s) nor risk factors can be identified as correlates to cerebral palsy 82) A patient with diabetes bring his glucose dairy from the past 7 days for the NP to review and evaluate what changes should be made? DAY 1234567 AM 67, 52, 61, 48, 39, 68, 70 PM138, 161, 148, 168, 171, 142, 176 a. increase the PM dose of NPH insulin b. increase the PM dose of regular insulin c. decrease the PM dose of NPH insulin d. decrease the PM dose of regular insulin Explanation: NPH Human Insulin which has an onset of insulin effect of 1 to 2 hours, a peak effect of 4 to 6 hours, and duration of action of more than 12 hours. 83) which of the following patient is at highest risk of suicide? a. 65 y/o female b. 70 y/o male c. 35 y/o white divorce male d. 42 y/o single mother 27 Explanation: There is a common perception that suicide rates are highest among the young. However, it is the elderly, particularly older white males that have the highest rates. And among white males 65 and older, risk goes up with age. White men 85 and older have a suicide rate that is six times that of the overall national rate. 84) a 17 y/o male is brought into the clinic by his mother who says that is there are no medical consequences or risk known, she will allow her son to get a possible. How should the NP respond? a. tattoos distort one’s body image resulting in high possible of depression b. tattoos and body piercing are possible sources of hepatitis C infection if there is a common use of contaminated needles. c. the tattoo should be applied to an area that will not be exposed to sun to avoid increase risk of skin cancer d. the only medical consequence of tattoos is the scarring associated with removal. Explanation: Sharing personal care items that may have come in contact with another person’s blood, such as razors or toothbrushes. Having sexual contact with a person infected with the hepatitis C virus. Getting a tattoo or body piercing in an unregulated setting 85) A diagnostic finding of Hodgkin’s lymphoma is the presence of: a. granulocytopenia b. reed Sternberg cells c. lymphoblast d. Howell -lolly bodies Explanation: He or she will diagnose classical Hodgkin's lymphoma if abnormal cells called Reed-Sternberg cells are found within the lymph node. Removing a sample of bone marrow for testing. A bone marrow biopsy and aspiration procedure involve inserting a needle into your hipbone to remove a sample of bone marrow. 86) A child with type 1 diabetes mellitus brings in a glucose dairy indicating consisted morning hyperglycemia. How you differentiate the Somogy effect from the dawn phenomenon? a. experiment with a smaller insulin dose in the evening b. increase the insulin dose and liberalize the child's diet c. check HbA1C today and again in 3 months d. instruct the parent to monitor the blood glucose at 3 am Explanation: Dawn phenomenon is an early morning rise in plasma glucose. It indicates a need for increased insulin. The Somogyi effect is a rise in plasma glucose in response to hypoglycemia. It is usually accompanied by weight gain and hunger and is corrected by decreasing the evening insulin dose. A series of 3 early morning measurements of blood glucose will help differentiate between the 2 conditions. 87) practitioners working with physically active girls need to be aware of the "female athlete triad" in order to develop an effective pain for prevention, recognition and treatment. the components of the female athlete triad are: a. distorted body image, low self-esteem, and generalized anxiety. b. stress fractures, osteoarthritis and plantar fasciitis c. eating disorders, amenorrhea and osteoporosis 30 Explanation: traditionally, ante-partum care began as soon as the pregnancy was identified. The new recommendation is to evaluate nutritional and psychosocial needs, and asses’ medical problems before pregnancy occurs. 96) which of the following must be present for the diagnosis of bacterial vaginosis? a. presence of clue cells b. vaginal pH <4.0 c. presence of pseudo hyphae on HPF d. negative amine test Explanation: Demonstration of clue cells on a saline smear is the most specific criterion for diagnosing BV. Clue cells are vaginal epithelial cells that have bacteria adherent to their surfaces. 97) The patient Self Determination Act of 1991 requires all health care agencies receiving Medicare or Medicaid funds to give patients written information about their rights to make decisions regarding their medical care. A document which declares in advance what type of medical care a person wants ... he provided or withheld should he or she be unable to express his or her wishes is called: a. an advance directive b. a durable power of attorney c. informed consent d. a right to die statement explanation: The PSDA requires all health care agencies (hospitals, long-term care facilities, and home health agencies) receiving Medicare and Medicaid reimbursement to recognize the living will and power of attorney for health care as advance directives. 98) Thalassemia should be included in the differential diagnosis of an infant with microcytic anemia when the infant’s parents are of what ethnicity? a. Italian b. Swedish c. Japanese d. European Explanation: Thalassemia can affect both men and women. Certain ethnic groups are at greater risk: Alpha thalassemia most often affects people who are of Southeast Asian, Indian, Chinese, or Filipino descent. Beta thalassemia most often affects people who are of Mediterranean (Greek, Italian and Middle Eastern), Asian, or African descent. 99) What is the earliest age that MMR immunization can be administered? a. 2 months b. 4 months c. 6 months d. 1 year Explanation : Both vaccines are administered by the subcutaneous route. The minimum age for both MMR and MMRV is 12 months of age. The typical age for the second dose of either 31 vaccine is at 4 to 6 years of age. The maximum age MMRV for administration is 12 years of age. It should not be administered to anyone 13 years of age or older. 100) A male patient with chronic atrial fibrillation takes a genetic brand of Coumadin (Warfarin). He should report all of these to his health care provider EXCEPT a. Brand name substitution for his genetic warfarin b. On missed dose of warfarin c. Chest pain or weakness d. Blood in his urine Explanation : If you miss a dose of this medicine, take it as soon as possible. However, if it is almost time for your next dose, skip the missed dose and go back to your regular dosing schedule. Do not double doses. 101) which of the following is not an appropriate treatment for chronic bacterial prostatitis (CBP)? a) Doxycycline (Vibramycin) b) Ciprofloxacin (Cipro) c) Trimethoprim/sulfamethoxazole (Bactrim DS) d) Nitrofurantoin (Macrobid) Explanation: Appropriate treatment of chronic bacterial prostatitis include fluoroquinolones, sulfonamides, tetracyclines, an microlights.....drop entry into the prostate is limited. fluoroquinolone's and sulfonamides achieve the highest concentration In the tissue; tetracyclines and microlights can also reach therapeutic levels in the prostate. nitrofuran towing is avoided in men with prostatitis because of concern about poor tissue penetration and poor efficacy. there is also a risk of pulmonary an hepatic adverse effect with prolonged use. recurrences of chronic bacterial prostatitis are common in warrant a 2nd …. Duration. The antibiotics of choice are the fluoroquinolones (eg, ciprofloxacin, Trimethoprim/sulfamethoxazole. Trimethoprim/sulfamethoxazole, Azithromycin. It has good penetration into the prostate and is active against gram-positive bacteria, Fosfomycin. Fosfomycin achieves reasonable tissues. 102) A new patient presents to the NP clinic stating she wants a second opinion. She started propiluracil (PTU, Propyl Thyracil) 75 mg 3 times a day a week ago as therapy for newly diagnosed Grave’s disease. She still feels irritable and jittery. How should the NP response? a) Obtain a serum TSH and serum T4 today b) Increase the PTU dose to 100 mg 3 times a day c) Recommend radioactive therapy d) Inform the patient that improvement requires 2 – 2 weeks. Explanation: Propylthiouracil should have some effect on your symptoms around three to four weeks after treatment starts. Your thyroid hormone levels should be stabilized within four to eight weeks. 103) A 15 y/o patient has a complaint of vaginal discharge. She is sexually active with multiple partners. Which of the following symptoms should lead the NP to suspect pelvic inflammatory disease (PID)? a. A report of dyspareunia 32 b. A complaint of low back pain c. A yellow vaginal discharge d. Cervical motion tenderness (CMT) Explanation : Cervical motion tenderness or cervical excitation is a sign found on a gynecological pelvic examination suggestive of pelvic pathology. Classically, it is present in the setting of pelvic inflammatory disease (PID) or ectopic pregnancy and is of some use to help differentiate PID from appendicitis. 104) Which of the following is NOT a component of the fetal biophysical profile? a) Gestational age estimate b) Fetal breathing c) Amniotic fluid volume d) Fetal movements Explanation: There are 5 components in the fetal biophysical profile: fetal heart acceleration, fetal breathing, fetal movements, fetal tone, and amniotic fluid volume measurement. 105) Which murmur is associated with radiation to the neck? a. Pulmonic stenosis b. Aortic stenosis c. Hypertrophic obstructive cardiomyopathy d. Mitral valve insufficiency Explanation: The classic murmur of aortic stenosis is a high-pitched, crescendo-decrescendo (diamond shaped), midsystolic murmur located at the aortic listening post and radiating toward the neck. The radiation of the AS murmur is often mistaken for a carotid bruit. 106) When examining a pregnant patient, where should the fundal height be at 22 weeks? a) Slightly above the symphysis pubis b) At the level of the umbilicus c) Midway between the symphysis pubis and the umbilicus d) Above the umbilicus Explanation: Between 18 and 32 weeks, there is good correlation between fundal height and gestational age of the fetus. The expected heights are: 10-12 weeks: fundus slightly above the symphysis pubis 16 weeks: fundus midway between the symphysis pubis and umbilicus 20 weeks: fundus at the level of umbilicus 28 weeks: fundus 3 fingerbreadths above the umbilicus 36 weeks: fundus just below the xiphoid process 107) Education of women with fibrocystic breast disease should include which of the following statement a. Fibrocystic breast disease is often a precursor of breast cancer b. Annual mammography is recommended beginning at age 40 c. Caffeine may trigger breast pain d. Oral contraceptives are not useful in the treatment of this disease 35 d. Insist that the victim leave the abusive environment immediately Explanation: Domestic violence survivors are at a high risk of developing post-traumatic stress disorder, substance abuse or stress-related mental health conditions. Learning to cope with residual emotional pain and fears is essential to healing. 116) A 60 y/o female patient complains of sudden unilateral, stabbing, surface pain in the lower part of her face lasting a few minutes, subsiding and then returning. The pain is triggered by touch or temperature extremes. Physical examination is normal. The patient had a negative dental examination. CT scan is normal. Erythrocyte sedimentation rate (ESR) is normal. Which of the following is the most likely diagnosis? a. Trigeminal neuralgia b. Temporal arteritis c. Parotiditis d. Bell’s palsy Explanation: Symptoms of trigeminal neuralgia include an acute onset of sharp, stabbing pain to one side of the face. It tends to begin at the angle of the jaw and radiate along the junction lines; The pain is severe and described as an electric shock. It may be made worse by light touch, chewing, or cold exposure in the mouth. In the midst of an attack, affected individuals shield their face trying to protect it from being touched. 117) A 45-year-old pre-menopausal female complains of indigestion, flatulence RUQ and epigastric “crampy pain” that radiates to the right scapula. Symptoms are exacerbated by a high fat meal. What is the most likely diagnosis? Chronic cholecystitis Explanation: the “typical” with chronic cholecystitis is “female, fat, ….. and ……40” year of age. Myocarditis produces retrosternal pain that is not related to diet. The pain of pancreatitis radiates to the back. The pain of ….and is near specific upper quadrant pain 118) Sings and symptoms of anemia, regardless of the specific cause or type, include all of the following EXCEPT: a. Weakness, dizziness , fatigue b. Shakiness and jitteriness. c. Dyspnea on exertion d. Shakiness or …..ness Explanation: Regardless of the cause or type anemia may often be mistaken for depression. All of the patient’s mental functions may be …. Generalized malaise, sluggishness, forgetfulness, lethargy, syncope, pallor, and problems associated with decreased tissue oxygenation. There is no associated nervousness or anxiety. 119) Advance directives are: Documents that guide medical decision making in the event an individual becomes incompetent or unable to convey his wishes. 36 Explanation: a document which designate authority to make health care decisions is a health care proxy or durable power of attorney, which is only one example of a type of advance directive. Another type of advance directive is a living will, which usually states the patient’s wishes not to receive life-prolonging treatment. Advance directives refer to several different kinds of documents that guide medical decision making. They ate recognized in some, but not all states. 120) The CAGE, MAST and AUDIT questionnaires to detect problem drinking should be used: a. To detect early problems and hazardous drinking b. As supplements to the standard patient history c. As a diagnostic tool for the early detection of problem drinking d. To determine the level and pattern of alcohol use Explanation: standard screening instrument such as CAGE, AUDIT, and MAST should be used only as a supplement to the…. Development. 121) Which of the following scenarios in a 75 y/o patient would be inappropriately managed in an outpatient setting (i.e. the patient should be referred for a hospital admission? a. Secondary hypertension b. Unstable angina c. Orthostatic hypotension d. Vasovagal syncope Explanation: Patients with unstable angina nearly always require hospitalization for monitoring, coronary artery lesion treatment, and/or medication management. The other situations do not warrant hospitalization and could be managed safety and effectively in the outpatient setting. Malignant hypertension is more likely to result in hospitalization than secondary hypertension. 122) An adult female present with a hordeolum. A topical antibiotic is prescribed. Which of the following instructions is NOT appropriate for the nurse practitioner to give the patient? a. Apply warm, moist compresses several times per day b. Do not use soap near the affected eyelid c. Do not wear eye makeup and discard all used eye makeup d. Do not rub your eyes Explanation: Good peri-orbital hygiene should prevent recurrence of the hordeolum. 123) The cornerstone of treatment for stress fracture of the femur or metatarsal stress fracture is Absolute rest from activities which may further stress the bone Explanation: absolute rest from aggravating activities is essential until healing has occurred. 124) Group A Beta hemolytic streptococcal (GABHS) pharyngitis is most common in which age group? a. Under 3 years of age b. Preschool children c. 6 to 12 years of age 37 d. Adolescent 40 131) A 55 y/o male patient complains of blood in his urine for the past 2 weeks. He denies dysuria. Urinalyses is positive for red blood cells and negative for leukocytes and nitrates. The initial evaluation of this patient should include: a. Magnetic resonance imaging (MRI) b. Cystoscopy c. Bladder biopsy d. Renal ultrasonography 132) A good sunscreen lotion or sunblock may contain all of the following components. Which provides the LEAST protection against the sun’s harmful rays? a. Zinc oxide b. Benzophenones c. Para-aminobenzoic acid (PABA) d. Lanolin Explanation: the most common sunscreens contain chemical filters. These products typically include a combination of two to six of the following active ingredients: oxybenzone, avobenzone, octisalate, octocrylene, homosalate and octinoxate. Mineral sunscreens use zinc oxide and/or titanium dioxide. 133) When examining 5 years old the NP knows that this is an appropriate age to teach the child about: a. Washing his hands before eating b. Not allowing anyone to touch his private parts without permission c. The importance of balanced meals so that he can grow tall and strong d. Getting his preschool immunizations Explanation: Anticipatory guidance by the NP should include an explanation about not allowing anyone to touch his private parts without permission. This is the age of sexual exploration for the child. 134) A mother presents with her 12-year-old son who has chills, fever, headache, malaise, and myalgias. On examination the NP finds an erythematous lesion, 7.5 cm diameter, with annular edges and central clearing. The boy first noticed the lesion when he was preparing to go back home from a 2-week summer camp in the mountains. Of the following, which is the most likely diagnosis? a. Pityriasis rosea b. Impetigo c. Lyme disease (erythema migrans) d. Tinea corporis Explanation: Lyme disease is caused by the bacterium Borrelia burgdorferi and rarely, Borrelia mayonii. It is transmitted to humans through the bite of infected blacklegged ticks. Typical symptoms include fever, headache, fatigue, and a characteristic skin rash called erythema migrans. 41 135) A NP is holding a prenatal nutrition class for a group of patients. Considering knowledge about cultural variations, which of the following women may be at increased risk for inadequate calcium intake? a. A 30-year-old orthodox Jew b. A 35 years-old Caucasian c. A 27 years-old Native American d. A 24 years-old Hispanic Explanation: While anyone can develop osteoporosis, Caucasian and Asian women are at higher risk than those of African American and Hispanic descent. 136) Which behavior would NOT be expected of a 2 y/o? a. Kicking a ball b. Running around a corner without falling down c. Riding a tricycle d. Climbing on a chair and standing up to reach and object Explanation: Stand on tiptoes. Kick a ball. Start to run. Climb on and down from furniture without help. Walk up and down stairs while holding on. Throw a ball overhand. Carry a large toy or several toys while walking. 137) A 49 years-old Hispanic female has a blood pressure of 145/95 mmHg during a routine annual evaluation. She has no previous history of hypertension. She takes a statin for dyslipidemia. How should the NP proceed with this patient? a. The patient’s blood pressure should be checked in about 2-4 weeks b. The patient should be started on 25 mg HCTZ (hydrochlorothiazide) once daily in the morning c. She should be diagnosed with hypertension today and lifestyle modifications should be initiated d. The statin should be stopped because it may be the cause of elevated blood pressure Explanation : Diagnosis of hypertension should be confirmed at an additional patient visit, usually 1-4 weeks after the first measurements. If white-coat hypertension is suspected. Treatment recommendation for specific patient population African American Patients should be treated with diuretics and calcium channel blockers first Those with chronic kidney disease aged 18-75 years should be prescribed an ACE inhibitor or an ARB Differences in ARB agents and diuretics: • Losartan (Cozaar): weakest ARB, may require twice a day dosing • Valsartan (Diovan), Telmisartan (Micardis), irbesartan (Avapro), candesartan (Atacand), Olmesartan (Benicar) are all genetics (except ebardi) can be used every day. • More Potent ARB include Olmesartan and azilsartan • Chlorthalidone is more potent than HCTZ 42 For HTN that is difficult to control : • Make sure the Pt. is on Na restricted diet of 1500 -2000mg/day and NOT taking OTC NSAIDs or drinking excess alcohol • If the patient is already on ACE/ARB w/thiazide diuretic and SBP remains uncontrolled, 1st try changing the ACE inhibitor to an ARB; if the pt. is on losartan change to a more potent once daily • If the Pt. is no B-Blockers change for a vasodilating B-Blocker. • Add dihydropyridine Calcium Chanel Blocker. Do not use Nifedipine with LV systolic dysfunction • Add aldosterone Blocker if BP remains uncontrolled • Change HCTZ to chlorthalidone (more potent thiazide) For elevated isolated systolic BP • Use ARB, a thiazide diuretic and or dihydropyridine calcium channel blocker 138) A 37 y/o female patient with a history of a single episode of depression and frequent complaints of PMS is being treated for hypothyroidism. Today she complains of poor concentration and fatigue. Initially the NP should: a. Provide information about PMS b. Perform a TSH c. Resume her antidepressant medication d. Question her further Explanation: 139) When a patient presents with symptoms of acute gallbladder disease, what is the appropriate a. Order abdominal x-rays b. Order abdominal ultrasound c. Refer the patient to a surgeon for evaluation d. Prescribe a second-generation cephalosporin Explanation: Ultrasound testing uses sound waves to take images of the gallbladder. It is the gold-standard to look for gallstones because it is simple and non-invasiveComputed tomography (CT): This test uses X-rays to construct detailed images of the abdominal organs. 140) The client with iron deficiency anemia should be advised to take the iron supplement a. With milk to avoid stomach upset b. With milk of magnesia at bedtime to avoid constipation c. On an empty stomach between meals d. For 30 days to 6 weeks 141) A 2-week-old infant presents with projectile vomiting, weight loss, dehydration, constipation and history of insatiable appetite. An olive shaped mass is palpable to the right epigastrium. The NP, suspecting pyloric stenosis, refers the infant for an upper GI series. The expected finding is: a. Double bubble pattern, secondary to air in the stomach, and a distended duodenum 45 Explanation : High blood pressure can often be prevented or reduced by eating healthily, maintaining a healthy weight, taking regular exercise, drinking alcohol in moderation and not smoking. Healthy diet Cut down on the amount of salt in your food and eat plenty of fruit and vegetables. 147) A 50 years old patient has abnormal vaginal bleeding with heavy periods and intermenstrual watery discharge with a small amount of blood. What is the most likely diagnosis? a. Uterine fibroids b. Normal perimenopause c. Endometrial cancer d. Cervical cancer Explanation: In women with postmenopausal bleeding, likely causes of uterine pathology should be evaluated. in early menopause the most common etiology of bleeding is atrophy of the endometrium or vaginal mucosa. if the patient has been postmenopausal and bleeding occurs, the most common causes are polyps, fibroids an endometrial hyperplasia. Non-pharmacological interventions include lifestyle modifications like dietary modifications, exercise, avoiding stress, and minimizing alcohol consumption. 148) Which choice below is NOT a symptom of COPD? Fully reversible airflow Explanation: COPD is characterized by airflow that is not fully reversible. Asthma patients have reversible flow disease. 149) Therapeutic international normalized ratio (INR) for a patient taking warfarin (Coumadin) for chronic atrial fibrillation is expected to be: Increased Explanation: Warfarin is expected to increase the INR in a patient who is taking it. Therapeutic levels vary according to the reason anticoagulation is needed. Therapeutic INR for a patient with a medical heart valve is 3.5 to 4.5. A patient with chronic atrial fibrillation requires a therapeutic INR of 2.0 to 3.0. 150) A healthy 50-year-old patient presents to the clinic for a routine physical examination. He has no significant personal or family history of clinical heart disease. What should be part of his CV assessment? Blood pressure only Explanation: The Guide to Clinical Preventive Services, a Report of the U.S. Preventive Services Task Force, does not recommend screening with electrocardiogram (ECG) for asymptomatic men or women at age 50; or screening for asymptomatic AAA until age 65 years. BP screening should take place annually for all adults. Routine x-ray is not recommended for asymptomatic adults. Carotid auscultation is not recommended routinely because of its poor sensitivity and specificity for carotid artery disease. Routine screening should be based on patient history, patient risk factors, and likelihood of development of clinical disease based on patient risk factors. 46 151) The NP is initiating levothyroxine (Synthroid) for primary hypothyroidism in a 71- year- old female. The usual starting dose of levothyroxine is 50 (0.05 mg) to 100mcg (0.100mg) per day. What would be the most appropriate initial therapy in this patient? 25 mcg (0.025 mg) per day Explanation: A principle of pharmacotherapeutics in the elderly is “start low and go slow.” In older patients, 25 mcg per day is usual starting dose. Larger doses may precipitate tachycardia and chest pain. 152) An example of an indirect role of the nurse practitioner is: Educator, administrator, or researcher, influencing the delivery of direct care. Explanation: NP’s have traditionally delivered direct patient care. Since there are increasing numbers of nurse practitioners who are prepared at the graduate level of education, there is increasing need for indirect roles such as educator, administrator, consultant, and researcher. 153) In July 1996, the Advisory Committee on Immunization Practices (ACIP) of the Centers for Disease Control and Prevention (CDC) issued which of the following recommendations for the use of the live attenuated varicella vaccine? Varicella vaccine has been approved for routine use in all healthy children age 12 months to 12 years. Explanation: Varicella virus vaccine has been approved for use in healthy children age 12 months to 12 years. Children in this age group should receive one 0.5 ml dose of the vaccine subcutaneously. Serologic testing is not warranted. Vaccination is not necessary for children who have a reliable history of varicella. Persons 13 years-of-age and older should be. Given two 0.5 ml doses of the vaccine, 4 to 8 weeks apart. 154) A patient presents with a furuncle in his right axilla. The NP should prescribe an oral antibiotic and: frequent warm moist compresses. Explanation: Warm moist compresses may provide some relief from pain and promote draining. 155) Which of the following signs and symptoms is typical of hyperthyroidism? Heat intolerance Explanation: Frequent stools, heat intolerance, weight loss, and anxiety or nervousness are indicative of hyperthyroidism function. Constipation, cold intolerance, weight gain, lethargy, and impaired concentration are indicative of hypothyroid function. 156) What is the most common chronic condition in the elder population in the U.S? Arthritis Explanation: The most frequently occurring chronic conditions among the elderly are arthritis (48%), HTN (36%), heart disease (32%) and hearing deficits (32%), cataracts (17%), and diabetes mellitus (11%). 157) A 7-year-old male presents with a painless limp, antalgic gait, muscle spasm, mildly restricted hip abduction and internal rotation, proximal thigh atrophy, and slightly short stature. The most likely diagnosis is: Legg-Calve-Perthes disease. 47 Explanation: These are the classic signs and symptoms of Legg-Calve-Perthes disease, a local, self-healing disorder. The mean age of presentation is 7 years. Justification for treatment is prevention of secondary osteoarthritis and femoral head deformity. 158) A 73-year-old presents with a very sore, glossy, smooth, beefy-red tongue. This clinical presentation most likely reflects: Pernicious anemia, due to insufficient intrinsic factor Explanation: Hypothyroidism is associated with an enlarged tongue (macroglossia). Cracked corners of the mouth (cheilosis), is common in the elderly, particularly when the patient is without teeth or dentures. The treatment is application of a topical lubricant at regular intervals. Insufficient production of intrinsic factor by the gastric mucosa decreases vitamin B-12 absorption, causing pernicious anemia. This anemia commonly presents with a very sore, smooth, beefy-red tongue. 159) All patients with renal calculi should be taught that the best prevention is: Increased fluid intake Explanation: Vigorous fluid therapy around the clock is beneficial for all forms of renal calculi. Fluid intake should be sufficient to assure urinary output of 2 to 3 liters per day. Protein intake should be restricted with calcium stones. Alkalization of the urine is beneficial with uric acid stones. 160) A patient complains of “very puffy feet” since beginning a new medication 3 months ago. There is no underlying physiological cause for this condition. Which medication is most likely causing the problem? Nifedipine (Procardia, Adalat) Explanation: Mild to moderate peripheral edema is associated with arteriolar vasodilation and not necessarily due to left ventricular dysfunction. It is most commonly seen in patients taking felodipine and nifedipine. 161) The diagnosis of human papilloma virus (HPV) infection in males is usually made by: Clinical appearance Explanation: Appearance of human papilloma virus is usually sufficient for diagnosis in male or females. The presence of cervical HPV is diagnosed via Pap smear. Viral culture is not useful for diagnosing HPV. Tzanck smear is used to diagnose herpes infections. Application of potassium hydroxide (KOH) to the lesion is used to diagnose fungal infection. 162) A patient that has 20/80 vision acuity can see: At 20 feet what a person with normal vision can see at 80 feet. Explanation: Visual acuity is screened by using the Snellen eye chart Vision of 20/20 is considered normal. “Legally blind” is defined as 20/200 vision in the better eye with corrective lenses. A person with 20/40 has better vision that a person with 20/60 or 20/80 vision. 163) The reason beta-adrenergic blockers should be avoided in patients with diabetes is because they may: Mask symptoms of hypoglycemia 50 minor 51 lesions on one leg. The most appropriate intervention is to advise the mother to wash the lesions thoroughly and: Apply a topical antibacterial ointment to the lesions on both children and administer an oral antibiotic to the older child. Explanation: When impetigo lesions are few and minor, a topical antibiotic (e.g., mupirocin) is sufficient. Multiple lesions, particularly when they are confluent, usually require an oral antibiotic as well. Keeping the lesions covered with clothing reduces further trauma by scratching and reduces transmission of this highly communicable disease. 176) Screening for increased intra-ocular pressure or early glaucoma is: Best performed by an eye specialist Explanation: There is currently no reliable method for primary care practitioner to screen efficiently for glaucoma. Accurate screening is best performed by an eye specialist with access to specialized equipment. 177) Which of the following accurately describes the appropriate use of influenza vaccine? Influenza vaccine should be given annually to all persons 65 years or older, and to person 6 month or older, who are in chronic care facilities, or who suffer chronic cardiopulmonary disorders, metabolic disorder, hemoglobinopathies, or immunosuppression Explanation: Influenza vaccine should be administered annually to all person’s age 65 years and older, and to person 6 moths of age or older. Influenza vaccine is also recommended for those who suffer from renal dysfunction, and for health care providers caring for high risk patients. 178) A NP is examining the gluteal folds of a 6-month-old and notices a 7 to 8 cm bluish discoloration in the sacral area. The NP should: Consider this a normal finding Explanation: bluish discoloration found in the sacral or gluteal area is usually indicative of a hyper-pigmented nevus commonly referred to as a Mongolian spot. There is often a family history of nevi. 179) What medication would be best to relieve symptomatic wheezing in a patient who has asthma? Short-acting bronchodilator Explanation: The most rapid relief for this patient will be achieved by nebulizing a short-acting bronchodilator, termed a “rescue” medication. Relief can be achieved in 1-5 minutes. Long- acting bronchodilators usually take 15-20 minutes to achieve bronchodilation. Methylxanthines are not usually used for relief of asthma symptoms. 180) Strabismus is observed in a 13-month-old child. The most appropriate action for the NP to take is to: Refer the patient to an ophthalmologist Explanation: Any ocular deviation seen beyond 6 months-of-age is considered abnormal and the patient should be referred to an ophthalmologist for evaluation. 52 181) Which of the following findings would raise the NP suspicious of bulimia in a 17- year- old female? Scar on her knuckles Explanation: Scars on the knuckles from induced vomiting are often found on persons with bulimia nervosa. Hypokalemia may be present. Persons with anorexia nervosa are commonly emaciated, but those with bulimia nervosa are usually normal weight. Frequent vomiting causes erosion of dental enamel. 182) A new patient presents to the NP clinic stating she wants a second opinion. She started propylthiouracil (PTU) 75mg 3 times a day a week ago as therapy for newly diagnosed Grave’s disease. She still feels irritable and jittery. How should the NP respond? Inform the patient that improvement requires 2-3 weeks Explanation: Because anti-thyroid drugs block the synthesis of thyroid hormone, but do not interrupt the release of stored hormone, clinical improvement is delayed for 2 to 3 weeks. A euthyroid state can usually be expected in 4 to 6 weeks. 183) Health Maintenance Organization (HMOs) and Preferred Provider Organizations (PPOs) are: Managed care systems Explanation: Managed care systems are intended to integrate delivery of health care with financing of health care. This is typically done through a series of contracts with health care providers, diagnostic groups, and other support services. 184) Which of the following is a microcytic hypochromic anemia? Iron deficiency anemia Explanation: Iron deficiency anemia is a microcytic and hypochromic anemia. Folic acid is a macrocytic normochromic anemia. Pernicious (vitamin B-12 deficiency) anemia is a macrocytic, megaloblastic, and normochromic anemia. The anemia of chronic disease is normocytic and normochromic. 185) Which drug class is associated with elevated serum lipid levels? Thiazide diuretics Explanation: Thiazide diuretics given in a higher dose are associated with elevated serum lipids and glucose. In lower doses, these elevations are less likely. 186) A 16-year-old presents for a sports physical for football. The NP auscultates a diastolic murmur. It is a grade II/IV. He has no history of a murmur. The patient denies symptoms. What is the most appropriate action for the NP? Refer this patient to a cardiologist Explanation: A diastolic murmur is ALWAYS considered abnormal, even if not accompanied by any physical symptoms. Diastolic murmurs tend to be soft (I-III/IV are common) because the heart is resting during diastole. This patient should not be allowed to practice football until he is evaluated by a cardiologist. He will need an echocardiogram as part of initial treatment. 55 199) A 19-year-old sexually active female is being counseled by the NP about contraception. The NP is accurate when she tells the patient that a diaphragm: Increases the incidence of UTI Explanation: Use of a diaphragm for contraception is associated with an increased incidence of UTI. The arching spring type is easier to properly place than the flat spring type. The diaphragm can be inserted up to 6 hours before intercourse. The risk of contracting a sexually transmitted disease is decreased with diaphragm use. 200) A 2-week-old infant is brought to the NP clinic after a difficult delivery. The mother says the infant fusses when handled or picked up. On physical examination, the NP notes decreased movement of the right arm during the Moro reflex, and crepitus on palpation of the right clavicle. The diagnosis is fracture of the clavicle. The recommended management is: Instructions to the parents to handle the neonate gently. Explanation: A frequent complication of a difficult birth is fracture of the clavicle. Neonates generally require only gentle handling of the arm and shoulder for 3 to 5 weeks. Older children may require a sling. X-rays are not usually necessary. 201) A 4-year-old presents to the clinic with circumoral pallor and an intense red eruption on both cheeks which appeared last night. The child has low grade fever but no other symptoms. What is the most likely diagnosis? Fifth disease Explanation: The eruptive rash of Fifth disease appears on the cheeks and forehead and looks like the child has been slapped; hence, the name, “slapped cheek” disease. This is a viral infection that stars on the face, but by the next day appears as a maculopapular rash on the extremities. It finally spreads to the trunk and distal extremities leaving a lace-like appearance. A prodrome with malaise and fever occurs occasionally. 202) The most effective intervention (s) to prevent stroke is (are): Smoking cessation and treatment of hypertension Explanation: According to the U.S. Preventive Services Task Force, the most important modifiable risk factors for stroke are hypertension and smoking. Improved treatment of hypertension has resulted in greater than 50% age-adjusted reduction in stroke. 203) A 30-years-old female has varicose veins. These are: Usually diagnosed on clinical presentation Explanation: Varicose veins are not congenital, but are due to development of faulty valves, usually at the saphenofemoral junction. They are more common during and after pregnancy. Women become symptomatic during menses. No special diagnostic studies are needed to diagnose varicose veins; they are evident by inspection. However, diagnostic studies are conducted prior to any surgical treatments. 204) A 3-year-old has worn a new pair of plastic sandals at the beach. When her mother takes them off, she notices that the straps of the sandals have left red marks and the child complains of itching and burning in the area of redness. What is the most likely diagnosis and how should the nurse practitioner manage the problem? Contact dermatitis; the mother should not allow the child to wear theses sandals again. 56 Explanation: This is a case of classic contact dermatitis. When the skin is exposed to an irritant, the stratum corneum is disrupted, the underlying epidermis is injured, and an inflammatory reaction ensues. Plastic is a common precipitant of this kind of reaction. The child should avoid exposure to known skin irritants. 205) Which of the following is NOT a symptom of irritable bowel syndrome? Weight loss Explanation: Irritable bowel syndrome (IBS) is characterized by cramping abdominal pain with painful constipation and /or diarrhea. Bleeding, fever, weight loss, and persistent severe pain are indicative of other problems. 206) After a thorough examination, a 2-month-old infant is diagnosed with infantile colic. What should be included in the initial education of the mother about this condition? Instruct the mother on appropriate feeding techniques. Then reinforce efforts to claim and comfort the infant. Explanation: There is no “cure” for infantile colic. The situation can only be managed until it resolves on its own. Effective management includes appropriate feeding technique, calming strategies, and patient support. Dicyclomine (Antispas), hyoscyamine (Lesvin), and simethicone (Mylicon) have indications for treatment of infantile colic. 207) Of the following signs and symptoms of heart failure (HF), the earliest clinical manifestation is: Weight gain Explanation: The earliest and most sensitive clinical indicator of HF is weight gain. A patient with HF should be instructed to weigh himself daily to note changes in his weight. The best time of day to weigh is early morning. The other clinical manifestations listed will present after the weight gain. Another early, clinical manifestation of CHF not listed above is fatigue. 208) After a thorough history, physical examination, and laboratory tests, a patient is diagnosed with irritable bowel syndrome. Which of the following initial treatment plans is currently considered most effective? A low-fat, tyramine-free, caffeine free, high fiber diet, along with a daily diary, and attention to psychosocial factors Explanation: Irritable bowel syndrome can be diagnosed with careful history and physical examination, along with complete blood count (CBC) to assess for anemia, and a biochemical profile and urinalysis, for general information. Invasive procedures should be avoided unless age or family history indicates risk. A low-fat, tyramine free, caffeine free diet, along with a daily diary and stress reduction are mainstays of therapy. Unresponsive patients should be referred to a gastroenterologist. Selective serotonin reuptake inhibitors (SSRI) may worsen symptoms by their excitatory effect on the bowel. 209) A 2-week old African American male infant has ecchymotic-like marks over his lower back and upper buttocks. The most appropriate intervention is to: Reassure the infant’s mother that this is a normal finding 57 Explanation: Mongolian spots are normal variants in non-Caucasian infants (particularly African American and Asian infants). They appear as “bruised” areas on the lower back and buttocks and usually fade spontaneously. 210) An elderly patient is taking an effective dose of doxepin (Sinequan) for treatment of agitated depression with insomnia. Constipation has become a significant problem, even though the patient has been vigilant about maintaining adequate hydration and uses bulk laxatives frequently. Which of the following is the course of action most likely to be successful? Stop the doxepin and initiate trazodone (Desyrel) Explanation: Fluoxetine tends to contribute to insomnia. A daily enema may cause fluid and electrolyte imbalance. Trazodone is a good alternative. It is a sedating antidepressant with properties similar to doxepin, but without anticholinergic effects. Constipation often accompanies depression and should be treated. 211) Which of the following is NOT true about Carpal Tunnel Syndrome? Phalen’s and Tinel’s signs are negative Explanation: Phalen’s and Tinel’s signs are positive and diagnostic of Carpal Tunnel Syndrome. Another hallmark feature is nighttime awakening with symptoms. Symptoms follow the distribution of the median nerve which innervates the first 3 fingers and the thumb-side half of the 4th finger. There is no involvement of the 5th finger. 212) A 17-year-old female presents with painful vesicular lesions on her vulva. Which of the following would be the most definitive diagnostic test? Tzanck prep Explanation: Tzanck prep is the only test in this which is diagnostic for herpes simplex. KOH prep is used to diagnose candida and bacterial vaginosis. Gram stain is used to help distinguish Gram positive and Gram-negative organisms. The Papanicolaou (Pap) smear is used to screen for cervical dysplasia and cancer. 213) Which of the following findings are usually associated with atopic dermatitis in infants? Positive family history Explanation: Atopic dermatitis is a familial disorder with unknown etiology. It is intensely pruritic. In infants it presents on the cheeks, forehead, scalp, trunk or extremities. It is often associated with asthma and allergy. 214) Today, a patient is diagnosed with iron deficiency anemia and started on a daily iron supplement. In order to best assess the adequacy of supplementation, the NP would appropriately order a follow-up: b. Hemoglobin level 1 one month Explanation: One month after iron supplementation, a hemoglobin or hematocrit should be measured to assess response to therapy. If this value is not increased in 1 month, consideration must be given to other sources of iron loss like GI or menstrual bleeding. The serum ferritin level, an indicator of tissue iron stores, should be increased and near normal after 4-6 months of supplementation. The reticulocyte count wound be expected to increase be 10 days or less but would not specifically assess the effects of iron supplementation. 60 the presence of a malabsorption disorder, inflammatory bowel disease, intolerance to oral iron, or blood loss too great to be compensated by oral iron. 223) The initial management of an adult with hypertension should include: evaluation for underlying causes explanation: approximately 5% of patients with hypertension in the US half secondary hypertension coma that is, it is secondary to something. Patient history, physical exam, and laboratory tests should be completed to identify patients who might have underlying causes (secondary hypertension ). underlying causes include some forms of renal disease, medications (oral contraceptives), adrenal hypertension, Acromegaly and hypercalcemia . 224) Which of the following statements about medication in a 45 years old patient with diabetes is correct? daily aspirin therapy should be recommended for the secondary prevention of a cardiovascular disease. Explanation: daily aspirin therapy for the secondary prevention of cardiovascular disease is recommended for patients with diabetes. The use of as the primary prevention, specially in low risk patients, is not routinely recommended. ACE inhibitors are especially beneficial right after albuminurea is present. Even if blood pressure is normal, an ace inhibitor may be used for renoprotection. The initiation of statin the dependent on cardiovascular risk or the presence of cardiovascular disease. ACE inhibitors may not be required with all patients with d…. 225) A 25 years old female has a history of frequent candidal vaginal infections in the past year. She is in a monogamous sexual relationship and uses an intrauterine for contraception. of the following, which is the most likely underlying condition for disposing her to recurrent candida vaginitis? a) hyperthyroidism b) diabetes mellitus c) trichomoniasis d) pregnancy Explanation: Diabetes mellitus (DM) is a metabolic disorder that predisposes individuals to fungal infections, including those related to Candida sp., due to a immunosuppressive effect on the patient 226) It is recommended that the therapeutic management of children with juvenile rheumatoid arthritis shoot include a) avoidance of exercise b) immunosuppressing agents c) ophthalmologists examination at least annually d) avoidance of aspirin therapy Explanation: Children with systemic in polyarticular JRA should have an annual ophthalmological examination. Those with pauci-articular JRA should be examine 4 time a year. 61 227) Stress urinary incontinence is: a) associated with the normal aging process b) may be caused by anti cholinergic or anti depressant medication c) due to the truther muscle instability d) might be aggravated by caffeine or alcohol Explanation: stress urinary incontinence is not expected as a result of the normal aging process. The primary problem is sphincter incompetence ingestion of caffeine or alcohol decreases sphincter control. Anticholinergic and antidepressant medication are causative factor overflow incontinence. Detrusor muscle instability is the primary underlying problem causing urge incontinence. 228) Which immunization is contraindicated in an immunedeficiency individual? 1. Varicella, 2. IPV, 3. MMR, 4. HBV a) 4 b) 1,3 c) 1, 3, 4 d) all of the above Explanation: People who are immunocomprised are at risk of adverse events or vaccine-related disease if they receive a live vaccine. Live vaccines include: • BCG (bacille Calmette–Guérin) vaccine • Japanese encephalitis vaccine • MMR (measles-mumps-rubella) vaccine • rotavirus vaccine • oral typhoid vaccine • varicella vaccine • yellow fever vaccine • zoster vaccine 229) Enlargement of the scrotum and testis with little change in the size of the penis characterizes which Tanner stage sexual development ? a) Tanner 1 b) Tanner 2 c) Tanner 3 d) timer 5 62 230) the most commonly recommended method for prostate cancer screening in a 55 y/o man is: Prostate-specific antigen (PSA) testing alone Explanation: a prostate-specific antigen (PSA) test is the most commonly recommended method for prostate cancer screening. For patient should … informed decision about whether to 65 indications for chronic HF treatment. Diuretics reduce pre-load, reducing such symptom, shortness of breath, orthopnea, and paroxysmal nocturnal dyspnea. 241) A 19 y/o pregnant patient, at 20 weeks gestations, complains of pain in the right lower quadrant . she is afebrile and denies nausea vomiting. The most likely diagnosis is: Round ligament pain Explanation: Ask the fetus grows and the uterus rises is out of the pelvis, the strain is placed on the you train round ligament causing lower abdominal pain. this common discomfort in pregnancy is not associated with any other systemic symptoms . 242) At 23 years old female present with his scaly hypopigmented macular lesion on her trunk, shoulders, an upper arms. The lesion fluorescent under the wood lamb. appropriate treatment for this condition is: Apply selenium sulfide (Versel) lotion and allowed to dry for 10 minutes; then rinse off. Explanation: take condition described is tinea versicolor, a fungal dermatitis. the lesions will have a Golden Florescence under Woods lamp. KOH Positive for hyphae and spores. distribution of the lesions is primarily on the chest, back and shoulders and less commonly the groin an genitalia. The condition typically presents in young adults and trends to be recurrent because the causative agent, Malassezia Normal skin inhabitant 243) A child complains of “hurting a lot” with swallowing and state that his “throat feels full”. his sore throat that started 4 days ago. he is hyperextending his neck. examination reveals asymmetrical swelling of the tonsils. his uvula is deviated to the left .what is the most likely diagnosis ? Peritonsillar Abscess Explanation: This can be a complication of pharyngitis due to failure to seek treatment or a failure to treat. peritonsillar or retropharyngeal Abscess is a reason for immediate physician or emergency 244) 15 years old pregnant patient present to her first prenatal visit. on an physical examination, her uterus is approximately 24 weeks growth in size. she doesn't know when her last menstrual period was nor does she know when she might have conceived. Gestational age for this patient can most accurately be assessed by? Ultrasonography Explanation: prior to the 26th week of pregnancy, ultrasound will accurately estimate just stational age with an accuracy within days. between the 14 and 30 weeks of pregnancy, measurement of fundal height is accurate within 2 weeks of the estimated gestational age . Naegele’s would not be accurate due to the lack of this patient menstrual history. Gestational age estimation is not a component of the biophysical profile. 245) Which APRN is the exception to the graduate level preparation requirement for certification for advanced practice registered nurses? Advanced practice registered nurses who have completed an approved ducational program prior to implementation of graduate level education are considered to have met the requirements for advanced practice registered nursing. 66 Explanation: Some advanced practice registered nurses in current practice have not been educated at the graduate level. Advanced practice registered nurses who have completed an accredited or approved educational program prior to implementation of graduate level education are considered to have met the educational requirement for advanced practice registered nurse (APRN) certification. 246) A 72 years old male present with fluid collected in his scrotum. He states that the size of his scrotum does not fluctuate during the course of a day. He is also likely to describe. A recent traumatic injury. Explanation: The condition described is a non-communicating hydrocele. This occurs about 1% of adult males and may be associated with infection, neoplasm, or trauma. The scrotum should translluminate if fluid is present. A solid mass will not transilluminate. The fluid may be surgically drained or monitored. 247) All of the following intervention with pediatric patient patients are appropriate EXCEPT: Pre-medicate all patient prior to all painful interventions. Explanation: Pre-medication is not warranted prior to all painful interventions. For example, pre-medication is not warranted to routine immunization, but it certainly is prior to suturing. Pain management should be an integral part of patient management. 248) Siri what is the recommendation of the US Department of Health and human resources regarding nicotine replacement therapy ? encourage the use of nicotine replacement except in the present of serious medical condition . Explanation: except in special circumstances such as pregnancy and recent myocardial infarction , every smoker should be offered nicotine replacement therapy . 6 month abstinence rates are 2 to 3 times higher for patients who use nicotine replacement as compared to placebo. 249) A patient takes Theophylline twice daily from Bronchospastic disease. today he presents with thick, discolored, tenacious sputum, fever and mild shortness of breath. what drug should he avoid? Clarithromycin (Biaxin XL) Explanation: This patient takes Theophylline, a drug Which is metabolized by the cytochrome P45O system. Biaxin XL Is also metabolized by the cytochrome P450 system. the addition of a second drug like Biaxin will slow down metabolism of the Theophylline causing the serum level to rise. Biaxin is not contraindicated in this patient but if it is prescribed, theophylline levels should be monitored closely and/or the dosage reduced. 250) Beneficiaries 21 years of age and older are allowed a specified number of outpatient visits per year. Explanation: In contrast to Medicare, Medicaid services are state specific. Each state received federal dollars to be used to take care of a group of individuals identified by each state. The individuals tend to be low income with no other payment source for medical expenses. 251) which group is considered to be high risk for the development of testicular tumors? Late adolescence through early adulthood. 67 Explanation: the peak incidence of tumors of the testes is in late adolescence and early adulthood; therefore, palpation of the testes is most likely to yield valuable information at this age. 252) a 45 y/o diabetic patient has periorbital cellulitis secondary to a sinus infection. What course of action should the nurse practitioner take? Consider collaboration with a physician regarding antibiotic treatment, culture, and subsequent action. Explanation: Periorbital cellulitis signals an emergent condition which may warrant hospitalization and intravenous antibiotics. At the very least, this patient should have a culture so that the organism can be identified and appropriate antibiotic can be administered. This patient is also diabetic, which contributes to his high risk status. 253) An adult female patient is seeking information about her ideal weight. she is 5 feet 7 inches tall. using the height weight formula, what is her ideal body weight? 135 pounds. Explanation: The height weight formula is a quick method of that remaining ideal weight. females allow 100 lbs for the first five feet of height plus 5 pounds for each additional inch. Males allow 106 pounds For the first five feet plus 6 pound for each additional inch. this method can only be used as an estimate because it does not account for body composition or age. 254) The mother of a 2 years old uncircumcised male patient is concerned that she cannot retract the foreskin over the glans penis. the appropriate response by the nurse practitioner is to? reassured the mother that this is normal Explanation: phimosis refers to 4 skin that cannot be retracted over the glans penies. this is a normal finding in uncircumcised male children Anne usually resolves by eight five years . circumcision is not indicated unless there is urinary obstruction. the foreskin should never be forcibly retracted. 255) After following, the patient who should be referred for periodic colonoscopy is the patient with: extensive ulcerative colitis of long duration Explanation: patients with extensive ulcerative colitis of long duration are at risk of colorectal cancer. Colonoscopic surveillance for these patients is recommended 256) A patient complaints of a stomach pains on and off for the past month in this team win in distinguishing between a gastric and duodenal ulcer, What question is least important to ask? have you been out of the country in the past several months? Explanation: responses B&C and D are critical questions to ask patients with suspected Pepcid ulcer disease. a positive response to response b implies a GI hemorrhage. A positive response to A could imply an infectious etiology of the stomach pain and has little bearing on distinguishing between gastric and duodenal ulcer disease. 257) The first step when taking a patient history is the: introductory information 70 266) The nurse practitioner diagnosis eczema on the cheeks of a 2 years old female. the patient's mother explained that she is embarrassed to take the child to public because her face is so red, dry, and crusted. she asked for a strong QWERTY sun cream so that eczema will clear rapidly. The nurse practitioner knows that: a) a high potency cortisone preparation applied to the face will shorten the duration of the eczema flare up. b) a high potency cortisone cream on the face will only be helpful if used in conjunction with hydrating lotion c) a high potency cortisone cream is not recommended for use in children's under 7 years of age d) A high potency cortisone cream might cause atrophy, telangectasia, purpura, or stanae if used on the face. Explanation: Topical steroids are grouped by potency into classes, from Class 1 (most potent) to Class 7 (least potent). Those in Classes 6 and 7 are usually most appropriate for children. Local side effects of topical steroids are more common and include: • Skin thinning (atrophy) and stretch marks (striae). • Easy bruising (senile purpura ) and tearing of the skin (fragility). • Telangiectasia . • Perioral dermatitis (erythematous papules around the mouth). • Contact allergy to the steroid molecule itself or another ingredient 267) An adult present with tinea corporis. what comment by the patient would be the least likely explanation of how he acquired this disease? a) I trap animals b) my wife has this also c) I have diabetes d) I live in the northern US Explanation: acquisition of infection may occur by direct skin contact with an infected individual or animal, contact with fomites, or from secondary spread from other sites of dermatophyte infection (eg, scalp, feet, etc). In particular, tinea corporis in adults may result from contact with a child with tinea capitis, often acquired by contact with an infected cat or dog. Tinea corporis can also occur in outbreaks among athletes who have skin-to-skin contact, such as wrestlers. Tinea corporis often begins as a pruritic, circular or oval, erythematous, scaling patch or plaque that spreads centrifugally. Central clearing follows, while an active, advancing, raised border remains. The result is an annular (ring-shaped) plaque from which the disease derives its common name (ringworm) 268) Which of the following can result from chronic inflammation of a meibomian Gland? a) A chalazion b) uveitis c) keratitis d) a pterygium 71 Explanation : A chalazion is a small, usually painless, lump or swelling that appears on your eyelid. A blocked meibomian or oil gland causes this condition. It can develop on the upper or lower eyelid 269) A 23-year-old female patient of Italian descent has been diagnosed with anemia secondary to glucose-6-phosphate dehydrogenase (G-6PD) deficiency. It is important to teach this patient to avoid: a) beef or pork liver b) fava beans c) phenylalanine d) milk and milk products Explanation: Glucose-6-phosphate dehydrogenase deficiency is a genetic disorder that occurs almost exclusively in males. This condition mainly affects red blood cells, which carry oxygen from the lungs to tissues throughout the body. In affected individuals, a defect in an enzyme called glucose-6-phosphate dehydrogenase causes red blood cells to break down prematurely. This destruction of red blood cells is called hemolysis. The most common medical problem associated with glucose-6-phosphate dehydrogenase deficiency is hemolytic anemia, which occurs when red blood cells are destroyed faster than the body can replace them. This type of anemia leads to paleness, yellowing of the skin and whites of the eyes (jaundice), dark urine, fatigue, shortness of breath, and a rapid heart rate. In people with glucose-6-phosphate dehydrogenase deficiency, hemolytic anemia is most often triggered by bacterial or viral infections or by certain drugs (such as some antibiotics and medications used to treat malaria). Hemolytic anemia can also occur after eating fava beans or inhaling pollen from fava plants (a reaction called favism). 270) A 62-year-old with Type 2 diabetes mellitus complains of increased nocturia, fatigue, and weakness. His fasting blood glucose is 110 mg/dL (6.2 mmol/L), he is slightly anemic, and his serum creatinine level is slightly elevated. All other laboratory tests and physical examination are within normal limits. What is the most likely diagnosis? a) Renal Insufficiency Explanation: Nocturia is a sign of moderate renal insufficiency due to impaired ability to concentrate urine. 271) A 12-year-old has a fasting serum cholesterol of 190 mg/dl (4.94 mmol/L). Her mother asks if this value is “ok”. The FNP responds: a) This is unacceptable. The desired level is <170 mg/dL (4.42mmol/L) Explanation: Children who have cholesterol levels greater than 170mg/dL (4.42mmol/L) are known to be at increased coronary artery disease. Total dietary intake of fats for children and adults should be < 30% of total caloric intake and < 7% should be from saturated fat. Reduction of dietary fat intake is not recommended for children <2 year-old-age. 272) The correct treatment for ankle sprain during the first 48 hours after injury includes: a) alternating heat and ice, and ankle exercises. b) resistive ankle exercises, ankle support, and pain relief. 72 c) rest, elevation, compression, ice, and pain relief. d) referral to an orthopedist after x-rays to rule out fracture Explanation: For the first 48 hours, elevate (raise) the ankle higher than your heart whenever you're lying down. If your ankle is just painful and swollen, remove ankle bracelets or toe rings right away. Then try the “RICE” method to ease your symptoms. RICE stands for “rest, ice, compress, and elevate.” Here’s how it works: Rest the ankle (use crutches if needed). Ice the ankle for 20 to 30 minutes every 2 to 3 hours for the first 2 days. Compress (wrap) the ankle lightly -- not tightly -- with an elastic bandage or ankle brace. Go to the emergency room if: Pain, bruising, or severe swellin. Bleeding, numbness, or change in color of the affected area. Deformed or seems dislocated. You can't put any weight on it 273) A 52-year-old female has a firm, non-tender, one-centimeter mass in the right lower quadrant of her breast. There are no palpable axillary lymph nodes. A mammogram the month before her examination was negative. The most appropriate NP action today is to: a) reassure the patient that the mass is benign b) refer the patient to a surgeon for evaluation c) schedule an ultrasound of the breast d) plan a repeat mammogram in 6 months Explanation: the diagnostic test of choice to differentiate a solid from a fluid-filled breast mass is ultrasound. More than 90% breast masses in women in the 20s to early 50s are benign. However, they must be evaluated. Clinical breast examination is unable to differentiate fluid filled from solid masses. MRI is not used unless a history of breast cancer is present. Mammography has the potential to evaluate the presence of a mass, but not adequate in assessing ii is fluid filled or not. 274) A 25-year-old married woman is being taught the natural family planning method (NFP) of contraception by the NP. Which of the following statements by the patient demonstrates her understanding of NFP? “Cervical mucus is clear and thin during ovulation.” Explanation: Cervical mucus is abundant, clear, thin, wet, and elastic, before and during ovulation. Increase in basal body temperature indicates ovulation. Abstinence is required when basal body temperature and cervical mucus assessment indicate ovulation. Specific dates of the cycle cannot be used with confidence. Douching can interfere with the ability to assess mucus changes. 275) A significant barrier to treatment of depression in the elderly population is: The belief that depression is an inevitable part of aging Explanation: Only 10% of elderly adults who suffer from depression receive treatment. This is largely because many health care providers and elderly patients mistakenly believe depression is an inevitable and untreatable result of ageing. 276) Oral and parenteral contraceptive methods: Inhibit secretion of FSH Explanation: The estrogen and progesterone in OCPs, and parenteral progesterone, inhibit pituitary production of folic stimulating hormone (FSH). 75 285) According to the American Diabetes Association, what is the lowest fasting plasma glucose level which warrants a diagnosis of diabetes Mellitus if confirmed on a subsequent day? a)121 mg/dL (6.7 mmol/L) b)126 mg/dL (7 mmol/L) c)130 mg/dL (7.2 mmol/L) d)140 mg/dL (7.8 mmol/L) Explanation: Glucose values that equal or exceed 126 mg/dL on different days constitute a diagnosis of diabetes. Therefore, an A1C is not needed for diagnosis but may be ordered to establish a baseline for this patient. Impaired fasting glucose can be diagnosed when two glucose values are between 100 mg/dL and 125 mg/dL on different days. 286) Which of the following patients is most likely to have a diagnosis of Type 2 diabetes mellitus? a) An underweight 9-year-old with frequent staphylococcal infections, glucosuria, and ketonuria b) A 49-year-old male with impotence and HbA1C 10.0% c) A 35-year-old obese female with frequent vaginal yeast infections and 120 fasting serum glucose d) A 55-year-old female with HTN, hyperlipidemia, and 130/dL 2 hour/75 g glucose tolerance test Explanation: The risk of type 2 diabetes increases as you get older, especially after age 45. That's probably because people tend to exercise less, lose muscle mass and gain weight as they age. Diabetes mellitus (DM) is a major risk factor for the development of erectile dysfunction (ED). 287) Which of the following are the classic features of ulcerative colitis? a) Right lower quadrant pain, frequently accompanied by a palpable mass, fever, and leukocytosis b) Massive painful hematemesis, occasionally accompanied by melena c) Rapidly progressive dysphagia to solid foods, anorexia, and weight loss out of proportion to the dysphagia d) Remissions and exacerbations of bloody diarrhea, tenesmus, fecal incontinence, abdominal pain, and weight loss Explanation: ulcerative colitis is a idiopathic inflammatory condition that presents with diffuse inflammation of mucosa in the colon, affecting the rectum upward. The hallmark sign is bloody diarrhea. Patient with severe symptoms may present with severe anemia, hypovolemia and impaired nutrition due to having more than 6 bloody bowel movements per day. 288) A 37-year-old female is found to have a negative rubella titer. How long after immunization should she avoid pregnancy? a) 28 days b) 60 days c) 90 days d) 120 days Explanation: Women of childbearing age should be advised to avoid pregnancy for 28 days after immunization with an MMR. However, The CDC has collected data on women who have 76 accidentally received that immunization while pregnant, Anne there has been no documented injury to offspring. the vaccine is safe for women who are breast feeding even though their rubella virus is excreted in breast milk. it is safe for young children of pregnant women to be immunized with NMR because infection is not transmitted from immunized patience. 289) Successful management of a patient with attention deficit hyperactivity disorder (ADHD) is best achieved with: a) Stimulant medication along with behavioral and family interventions b) Methylphenidate (Ritalin) in conjunction with an antihistamine c) treatment by a pediatric psychiatrist d) firm discipline and removal of offending foods from the diet Explanation: Pharmacotherapy and behavioral therapy are currently used with success in treating attention-deficit/hyperactivity disorder (ADHD) in children, adolescents, and adults. 290) Which of the following signs and symptoms are typical of hypothyroidism? a) Constipation b) Heat intolerance c) Weight loss d) Nervousness Explanation : Hypothyroidism happens when your thyroid gland doesn’t make enough hormones. This can slow down many of your body functions, including your bowels. 291) A first-time mother asks the NP how much her baby will grow in the first 3 months of life. The NP explains that the normal baby will grow. a) ½ to 1 ounce per day b) 1 to 2 ounces per week c) one inch per week d) more than 6 inches Explanation: Once your milk supply is established, your baby should gain between ½ and 1 ounce per day during the first 3 months 292) The requirement that a NP practice under the direct supervision of a physician is: a) Not a requirement in any state b) Dependent on state regulations c) Illegal, according to standards of nursing care d) A standard national requirement Explanation : The central principle underlying physician supervision of NPPs is that the physician retains ultimate responsibility of the patient care rendered when so required by state law 293) The NP correctly diagnoses iron deficiency anemia in a male patient whose lab report reveals: a) hemoglobin (Hgb) 15.0 g/dL (150g/L) b) a decreased total iron-binding capacity (TIBC) 77 c)mean corpuscular volume (MCV) < 80 fL d) increased mean corpuscular hemoglobin concentration (MCHC) Explanation: In iron-deficiency anemia, red blood cells will be small in size with an MCV of less than 80 femtoliters(fl) 294) All of the following are medical emergencies which may be attributed to acute cocaine intoxication EXCEPT: a) hyperthermia leading to extreme rhabdomyolysis b) HTN with or without vasculitis causing cerebrovascular accident (CVA) c) depression of cardiac conduction and contractility resulting in arrhythmias and myocardial infarction (MI) d) decrease heart rate and vasodilation leading to hypersomnia Explanation: Cocaine is associated with both ischaemic and haemorrhagic stroke. Persistent severe hypertension in the presence of chest pain. We conclude that cocaine intoxication can cause acute rhabdomyolysis with acute renal failure, severe liver dysfunction, and disseminated intravascular coagulation. 295) A male patient presents 24 hours after having been bitten on the arm during a fight with his spouse. His wound is swollen, red. Has a discolored exudate and a 5-centimeter tissue scar. Which action by the NP is LEAST appropriate at this time? a) Administer 0.5 ml of tetanus toxoid vaccine since it has been > 5 years since his last tetanus booster b) Prescribe amoxicillin-clavulanate (Augmentin) and collect a swab specimen for culture and sensitivity c) Suture the torn tissue since 24 hours has elapsed since the injury. Provide an antibiotic ointment for use with twice a day dressing changes d) Prescribe an antibiotic ointment for application with twice a day dressing changes. Explanation: Wound closure is a source of controversy in the management of patients with human bite wounds. In general, do not close hand wounds, puncture wounds, infected wounds, or wounds more than 12 hours old. Allow such wounds to heal by secondary intention. They may be closed secondarily or revised at a later date. 296) Adolescents are at increased risk for contracting the human immunodeficiency virus (HIV) due to: a) immaturity of their immune systems b) increased sexual experimentation c) pubertal changes in body composition d) inadequate primary health care Explanation: Teenagers need to be educated about how HIV is spread and ways to prevent transmission because the virus is spread by sexual contact and intravenous drug use. Teenagers are likely to take part in these risky behaviors because of their developmental stage. 80 Explanation: Diverticulitis is an inflammation or infection of the pouches formed in the colon (diverticula). Can last several days or weeks. Common ages are 35-50. Requires lab test and imaging. 304) At what age does vision normally become approximately 20/20? a) 6 weeks b) 4 months c) 3 to 4 years d) 6 years Explanation: 4-6 years of age, child recognizes and recites the alphabet, ready to read, has complete depth perception, and should have 20/20 vision. 305) Salmeterol (Serevent) in combination with an inhaled steroid is prescribed for a patient with moderate persistent asthma. What is the most important teaching point about salmeterol? a) It is not effective during an acute asthma attack b) It may take 2-3 weeks to begin working c) This drug works within 10 minutes d) This drug may be used by patients 6 years and older Explanation: This medication is not to be used to treat intense flare-ups or shortness of breath. A rescue inhaler will be needed. 306) At what age is screening most helpful in detecting scoliosis? a) 4 to 6 years b) 8 to 10 years c) 12 to 14 years d) 18 to 20 years Explanation: girls should be screened twice at 10-12 yrs of age and boys ages 13-14 307) A first time mother asks the NP when she should start feeding her baby cereal. The NP replies: a) not before 4 to 6 months-of-age b) at 2 months of age if the baby is taking 20 ounces of milk per 24 hours day c) when the baby does not sleep through the night d) as soon as the baby begins teething Explanation: The Academy of Pediatrics does not recommend introducing cereal and other solid foods until 4-6 months of age. Before that, only breast milk or formula. 308) Which symptoms is NOT typical in a female during the peri-menopausal period? a) Vasomotor instability b) Paresthesia c) Increased vaginal lubrication d) Disturbed sleep patterns Explanation: Females in the peri-menopausal period have decreased vaginal lubrication which results in itching, discharge, bleeding, and increased risk of urethritis. Vasomotor instability 81 ("hot 82 flashes" and "hot flushes") is the most common symptom during this period. Other symptoms include nervousness, palpitations, depression, and headache. 309) An 85-year-old is diagnosed with shingles. The patient states that she became “miserable”. What is the pharmacologic interventions should the NP offer this patient? a) NSAIDs for pain and fever b) Topical capsaicin cream (Zostrix) to lesions 4 times a day for 21 days c) Oral acyclovir (Zovirax) for 7 to 10 days, NSAIDs, and topical capsaicin d) Acetaminophen, Burrow’s solution compresses and mupirocin Explanation: Medications include, antiviral drug, NSAIDS, analgesic or nerve pain medication. Capasaicin can relieve topical pain 310) The mechanism by which nurses are held accountable for practice, based on the quality of nursing care in a given situation with established standards of practice, is: a) outcome criteria b) process criteria c) peer review d) quality assurance Explanation: evaluation of scientific, academic, or professional work by others working in the same field. 311) A patient taking levothyroxine is being over-replaced. What condition is he at risk for: a) Osteoporosis b) Diarrhea c) Bipolar disorder d) Periorbital puffiness Explanation: Symptoms of levothyroxine being over-replaced will be: Increased pulse and bp, anxiety, nervousness, diarrhea, difficulty sleeping, feeling that your heart is racing/skipping beats, unexplained weight loss, increased appetite. 312) What prophylaxis medication is recommended for the patient under 35-years-of-age with…. a) ethambutol (Etibi) b) Streptomycin c) pyrazinamide (Tebrzid) d) isoniazid (INH) 313) The infant 1 to 6 months-of-age with a diagnosis of developmental hip dysplasia is correctly treated with: a) closed reduction of the hips b) surgical open reduction followed by pelvic and/or femoral…. 85 318) Which drugs below would be expected to produce the least amount of hypokalemia? a) Furosemide (Lasix) b) Hydrochlorothiazide (HCTZ) and spironolactone (Aldactone) c) Spironolactone (Aldactone) d) Hydrochlorothiazide (HCTZ) Explanation: Potassium-sparing diuretics such as spironolactone, amiloride, and triamterene all have the potential to cause hyperkalemia. This risk is increased when used in association with potassium supplements and salt substitutes, as previously noted. The risk of hyperkalemia with spironolactone increases threefold if used with potassium supplements. The use of concurrent loop diuretics does not guarantee that hyperkalemia will not occur 319) A 25-years-old overweight patient presents with a complaint of dull achiness in his groin and history of palpable lump that “comes and goes”. On physical examination, the NP does not detect a scrotal mass. The scrotum does not transluminate. Considering these finding what is the most likely diagnosis? a)Testicular torsion b) Epididymitis c) Inguinal hernia d) Varicocele Explanation: An inguinal hernia is characterized by herniation of bowel or omentum into the scrotum. It typically present with the scrotal pain and scrotal mask or scrotal swelling. Abdominal or growing pain with a scrotal mass is a common presentation. bowel sounds might be audible in the scrotum. 320) Acute idiopathic thrombocytopenia purpura (ITP) is most commonly seen in childhood following: a) trauma b) glucocorticosteroid therapy c) aspirin ingestion d) acute infection Explanation: Often, the child may have had a virus or viral infection about three weeks before developing ITP. It is believed that the body, when making antibodies against a virus, "accidentally" also made an antibody that can stick to the platelet cells. The body recognizes any cells with antibodies as foreign cells and destroys them. That is why ITP is also referred to as immune thrombocytopenic purpura. 321) Which of the following drug classes may potentiate hyperkalemia in a patient taking a potassium-sparing diuretic or potassium supplement? a) Thiazide diuretic b) Angiotensin-converting enzyme inhibitor c) Beta-adrenergic blocker d) Calcium channel blocker Explanation: ACE inhibitors work in the kidney in the renin angiotensin aldosterone system and can impair renal excretion of potassium in patients with normal kidney function. In patients with impaired renal blood flow and/or function, the risk of hyperkalemia is increased. Common 86 practice is to monitor potassium, BUN, and Cr at about one week after initiation of an ACE inhibitor and with each increase in dosage in a patient who has heart failure and who receives an ACE inhibitor. 322) The major health risk(s) associated with oral contraceptive therapy is (are): a) dysmenorrhea b) cardiovascular events c) infertility d) ovarian cancer Explanation: 323) A 35-year-old women with prior history of extreme nervousness has been treated with diazepam (Valium) for 4 weeks. She shows improvement. Today her pulse is 112 bpm and BP is 130/92 mmHg. She is 5’4”and weight 105 pounds. What is the diagnosis? a) anxiety b) somatization c) Cushing’s syndrome d) hyperthyroidism 324) Of the following characteristics, which are most closely associated with risk for becoming an abusive parent? a) older, following the example set by the previous generation b) young, isolated, with unreasonable expectations of the child c) working full time, feeling pressure to perform multiple duties d) single, working part time, living with parents, attending trade school Explanation: Parents with a lack of social support, a tendency toward depression, multiple stress factors, and a history of abuse are at risk for abusing their own children. Abusive parents often have unrealistic expectations of a child's developmental abilities. 325) A 55-year-old male patient presents with dysuria, urgency, perineal pain, and temperature of 101 degrees. What is most likely diagnosis? a) Cystitis b) Epididymitis c) Urethritis d) Prostatitis Explanation: Findings of acute bacterial prostatitis include fever, chills, in low back pain, as well as the dysuria, increased urgency or frequency of urination an nocturia. epididymitis usually uppers with scrotal edema and pain. 326) Licensure is: a) another term for certification b) contingent on certification c) used to establish minimal competence 87 d) necessary for reimbursement 90 d) penicillin 91 Explanation: Clarithromycin is an antibiotic recommended by the American Thoracic Society for treatment of community acquired pneumonia (CAP) in young adults without co-morbid conditions. Clarithromycin provides antimicrobial activity against Streptococcus pneumoniae, the most common causative organism in CAP, and provides coverage for other less common pathogens, including Mycoplasma pneumoniae, Chlamydia pneumoniae, and Legionella pneumophlia 335) Infants with celiac disease (gluten enteropathy) are at risk for multiple complications. The most urgent complication of this disease is: intussusception or volvulus Explanation: Intussusception and volvulus are surgical emergencies. Delay releasing the invaginated or “telescoped” bowel (intussusception) or releasing the twisted bowel (volvulus) may result in tissue death and gangrene, perforation, peritonitis, and/or sepsis, and fatality. There is also a high rate of intussusception and volvulus among infants with cystic fibrosis. 336) Prescriptive authority for APRN is: Legislative and regulated at the state level Uniform for all states Governed by the state medical board Regulated by federal and state laws
Docsity logo



Copyright © 2024 Ladybird Srl - Via Leonardo da Vinci 16, 10126, Torino, Italy - VAT 10816460017 - All rights reserved